Космос и астрономия

Ответить в тред Ответить в тред
Check this out!
Тред тупых вопросов №137 Laplace edition Аноним 07/09/20 Пнд 16:11:35 5994071
Pierre-Simon-ma[...].jpg 568Кб, 1429x1600
1429x1600
ProtoplanetaryD[...].jpg 1664Кб, 3000x2000
3000x2000
out.mp4 6622Кб, 1280x720, 00:00:24
1280x720
time-long-21500.jpg 144Кб, 1500x324
1500x324
Тред вопросов о жизни, Вселенной и всём таком.

Спрашиваем то, за что в других местах выдают путёвку в биореактор. Здесь анонимные учёные мирового уровня критически рассмотрят любые гениальные идеи и нарисованные в Paint схемы.

Предыдущий тут: >>596078 (OP)
https://2ch.hk/spc/res/596078.html

Q: Можно быстрее?
A: Можно упасть в пузырь Альбукерке, NASA уже почти надула его.

Q: Я начитался охуительных историй про уфологию, че делать, нам жопа?
A: Да, тебе жопа, можешь сгонять в зогач или куда оттуда пошлют.

Q: Что будет с человеком в вакууме без скафандра / если он упадет на черную дыру / попробует ступить на поверхность газового гиганта/солнца?
A: Он умрёт.

Q: Почему бы не привязать ракету к воздушному шару или стартовать с горы?
A: Космос - это не как высоко, а как быстро, большая часть энергии ракеты уходит на разгон вбок.
Подробнее тут https://what-if.xkcd.com/58/ (английский) https://chtoes.li/orbital-speed/ (перевод)
Аноним 07/09/20 Пнд 16:19:59 5994112
>>599407 (OP)
Ёкарный срам...
Перво-наперво, Алькубьерре.
Далее, не рухнуть, а смастерить вокруг судна изнутри (в противном случае не станет синематографа).
Ну и третье, НАКА сгибает пространство на десятимиллионную часть, контролируя это сверхточными интерферометрами, до самого варп-привода здесь - как до Антарктиды лобстером.
Аноним 07/09/20 Пнд 17:17:09 5994183
ui-5b9b4a3a7d34[...].jpeg 93Кб, 900x600
900x600
>>599407 (OP)
>пузырь Альбукерке
Слышал только про кристаллы из Альбукерке
Аноним 07/09/20 Пнд 17:21:20 5994194
>>599418
Эт фигня. Про дожди из алмазов на газовых гигантах услышишь - офигеешь.
Аноним 07/09/20 Пнд 17:21:27 5994205
>>599407 (OP)
>3
А что я собственно вижу на видео?
Аноним 07/09/20 Пнд 17:22:58 5994216
>>599420
Там же белым по чёрному написано: формирование планетезималей в протопланетном диске.
Аноним 07/09/20 Пнд 17:25:26 5994227
>>599419
Слышал. И про водородный кисель на уране тоже. И про "живые" облака на Юпитере.
>>599421
Вокруг звезды?
Аноним 07/09/20 Пнд 17:26:13 5994238
>>599422
>Вокруг звезды?
Например, да.
Аноним 07/09/20 Пнд 17:29:16 5994269
>>599423
Крупных небесных тел

Наитупейший вопрос пришел в голову.
А вокруг галактики могут рождаться гигантские планеты что будут выходить уже на их артбиту?
Аноним 07/09/20 Пнд 17:30:41 59942710
>>599426
Нет, это уже звезды будут. Да и там предел массы — где-то 200 солнечных, дальше только черная дыра.
Аноним 07/09/20 Пнд 17:31:47 59942811
>>599426
Да, газики могут сами по себе собраться за пределами галактики.
Насчет того, будет ли это планета - тут пока наблюдений нет, но гипотетически почему бы и нет.
Аноним 07/09/20 Пнд 17:32:05 59942912
>>599427
ПЛАНЕТЫ МАССОЙ 200 СОЛНЕЧНЫХ?!
Аноним 07/09/20 Пнд 17:32:55 59943013
>>599429
Звезды.
Выше 70 масс юпитера - уже звезда.
Аноним 07/09/20 Пнд 17:39:33 59943114
>>599430
Т.е горит, шкварчит, пердит вспышками и ррряно фонит радиацией?
Аноним 07/09/20 Пнд 17:41:16 59943215
Аноним 07/09/20 Пнд 17:54:29 59943316
>>599432
Не понимаю. Предположим такая хрень набрала массу n-юпитеров из булыжников, льда и прочего не газового дерьма. То чему там полыхать? Я конечно думаю такое не возможно из за природы гравитации, мол скрепов не хватит(или и тут ошибаюсь?), но вдруг, что то смогло настругать такого исполина.

И да, получается если юпитеру добавить 70 таких юпитеров, то он загорится?
Блин,
Аноним 07/09/20 Пнд 17:55:05 59943417
>>599433
>чему там полыхать
Водороду.
Аноним 07/09/20 Пнд 17:56:14 59943518
>>599434
Потому что его просто дохуя и у него нет выбора кроме как пиздануть?
Аноним 07/09/20 Пнд 17:57:47 59943719
>>599435
Ага.
Большая часть материи во вселенной - водород.
Ты соберешь достаточную массу, ты в любом случае наберешь дохерища водорода.
Аноним 07/09/20 Пнд 18:04:36 59943920
>>599437
Т.е таких объемов водород под силой гравитации начинает греться? А потом пошла пьянка.
Полагаю, что это будет взрыв так как остальные элементы этого гипотетического объекта не особо дружат с горящим водородом и все разлетится обратно в космос.
Аноним 07/09/20 Пнд 18:06:11 59944121
>>599439
Все бурлит и перемешивается, а вот водород при этом может фьюзиться выдавая энергию. Как выгорит, пойдет гелий и так далее до коллапса на железе.
Аноним 07/09/20 Пнд 18:06:27 59944222
>>599439
Кекус максимус просто.
Аноним 07/09/20 Пнд 18:08:50 59944323
>>599439
>водород под силой гравитации начинает греться
Водород под силой гравитации начнет термоядерный синтез. Остальные элементы частично улетят, частично прореагируют еще раньше водорода, частично просто будут булькать в плазме.
>гипотетического объекта
Этот гипотетический объект ты можешь наблюдать на небе каждый день.
Аноним 07/09/20 Пнд 18:13:00 59944524
147658010814253[...].jpg 197Кб, 675x425
675x425
>>599443
>Этот гипотетический объект ты можешь наблюдать на небе каждый день
[Смеётся по-петербуржски]
Аноним 07/09/20 Пнд 18:16:01 59944625
>>599443
>Остальные элементы частично улетят
И образуют планеты из другой херни на орбите?
Аноним 07/09/20 Пнд 18:17:45 59944726
>>599446
>планеты из другой херни
В Юпитере дохера водорода, например.
Аноним 07/09/20 Пнд 18:18:21 59944827
ow-supernova.webm 9715Кб, 1280x720, 00:01:57
1280x720
>>599446
Не на орбите. Облако разлетается в ебеня, соединяется с другими облаками и образует другую звезду/несколько со своими протопланетарными дисками, и, да, со своими планетами.
Здесь останутся лишь угольки из белого карлика/НЗ/ЧД и останки планет со сдутой и частично захваченной атмосфере, обреченные на остатки существования во тьме до тепловой смерти вселенной.
Аноним 07/09/20 Пнд 18:18:30 59944928
>>599445
Я когда в меде учился, у нас анатомия в ВМА была, в ноябре. Утром в темноте под дождем в склеп с трупами, 4 пары там, в 4 часа в темноте под дождем домой, питаться прямо на месте чебуреками, купленными у финки, солнца не видел месяц. Я там некромантом стал.
Аноним 07/09/20 Пнд 18:19:28 59945029
>>599447
Ну это типа тот что не был приглашен на тусу жарится в плазме или именно тот что выплюнуло?
Аноним 07/09/20 Пнд 18:21:49 59945130
>>599450
Скорее всего не был приглашен в тусу, но какая-то часть была выплюнута (звездный ветер, вот это все). Но у него ж не спросишь.
Аноним 07/09/20 Пнд 18:25:14 59945231
>>599451
Планета-хикка, со своей оняме коллекцией фигурок планет из 79 штук....
Аноним 07/09/20 Пнд 18:25:29 59945332
>>599450
У тебя поначалу вся жижа скукоживается в блинчик, там тебе все что было изначально в тусе. Оно по идее примерно равномерно образуется в планетезимали, привет Молтону, а та что ближе к центру образует протозвезду.
Аноним 07/09/20 Пнд 18:28:02 59945433
15994842966691.jpg 1599Кб, 3000x2000
3000x2000
>>599453
Проще говоря пикрелейтед?
Аноним 07/09/20 Пнд 18:31:00 59945534
image.png 317Кб, 860x460
860x460
image.png 1565Кб, 1280x853
1280x853
image.png 370Кб, 678x542
678x542
image.png 1024Кб, 1440x900
1440x900
Аноним 07/09/20 Пнд 18:41:28 59945835
>Металлический водород возникает при больших давлениях (около миллиона атмосфер) и высоких температурах, когда кинетическая энергия электронов превышает потенциал ионизации водорода.
Кстати да, насколько я помню, металлы отличаются тем, что у них коммунизм электроны как бы общие для всех атомов и образуют как бы электронный газ. И поэтому они хорошо проводят ток, в частности.

Я к чему: значит ли это, что при больших (очень больших) давлениях и температуре любой неметалл переходит металлическое состояние?
Аноним 07/09/20 Пнд 18:46:36 59946036
>>599458
>при больших (очень больших) давлениях и температуре любой неметалл переходит металлическое состояние?
А они раньше не развалятся?
Аноним 07/09/20 Пнд 18:48:41 59946237
>>599460
Зачем? В звездах же не разваливаются, там наоборот синтез идет.
Аноним 07/09/20 Пнд 18:55:45 59946538
Аноним 07/09/20 Пнд 18:57:00 59946739
>>599465
Быстродополнение: высокая температура тут никак не помогает, наоборот, нужна низкая.
Аноним 07/09/20 Пнд 18:59:26 59947040
Аноним 07/09/20 Пнд 19:01:07 59947141
>>599462
Я неудачно выразился, я в общем имел ввиду, там ядерные реакции не начнутся? В любую сторону. Водороду-то хорошо, там в ядре кроме протона нет нифига.
Аноним 07/09/20 Пнд 19:06:04 59947642
>>599471
У водорода как раз одни из самых простых условий для термоядерного горения, не зря ж звезды с него начинают (ну еще там литий-6 что ли, специально обученые изотопы, но их мало). Так что не должны. Но вообще, вон выше поправили, что высокая температура как раз и не нужна. Нужно что-то типа Юпитера на стероидах: охуенное давление из-за тысяч километров вещества снаружи, но относительно нежарко (так, тысяч 20 кельвинов всего). А еще лучше его в холодильник засунуть, наверное.
Аноним 07/09/20 Пнд 19:37:04 59948243
Я таки добьюсь ответа на свой вопрос. Почему бы не сделать первую ступень ракеты в виде самолета с несущим фюзеляжем? Взлетает, выкидывает вторую ступень, садится на аэродром downrange. Не нужно топливо для ховерслэма, не опрокинется, на выкидные крылья тоже массу тратить не надо. Может потом заправиться и своим ходом долететь обратно на космодром. А то какой-то максимализм сплошной, либо запускаем ракету с боинга, либо тащим блядь весь самолет на орбиту.
Аноним 07/09/20 Пнд 19:39:10 59948344
>>599482
Для профита тебе нужен высотный гиперзвуковой самолет.
Сразу становится дохуя СЛОЖНА.
Как бы это странно ни звучало, проще сделать ракетную ступень, у тебя и так они будут присутствовать.
Аноним 07/09/20 Пнд 19:44:56 59948945
>>599483
Доо блядь. Сделать ракету в форме самолета сложно, а тащить самолет на орбиту, да еще и обвешанный теплозащитой - это норм идея, пупок не развяжется.
Аноним 07/09/20 Пнд 19:46:24 59949046
>>599489
Шаттл хуевая затея, к чему тут твоя ложная дихотомия? Я про него ничего не говорил.
Аноним 07/09/20 Пнд 19:48:22 59949247
>>599490
А я про шаттл и не говорю. Мало ли маняпроектов самолетного вида? Скайлон, венчюрстар, одни долбоебы вообще на вертолете на орбиту летать хотели.
Аноним 07/09/20 Пнд 19:53:10 59949448
>>599492
Ты говоришь о воздушном старте, а упоминаешь ССТО, это другая категория.
Я ж тебе пояснил, что для практичности воздушного старта надо набирать заметную скорость и высоту, это сгорание в атмосфере и гиперзвук. Такие вещи есть, но они одноразовые. Чтоб был смысл надо чтобы они были еще и осмысленно многоразовые и тащили дохуя нагрузки при этом.
Проще вынырнуть из атмосферы на каком-нибудь флаконе и байкале и вернуться на нем на старт, технологии все есть, никаких многоразовых ГПВРД и тепломатериалов разрабатывать не придется.
Ближе всего к твоей хотелке венчурстар, в целом классная затея, я бы хотел чтоб он взлетел.
Аноним 07/09/20 Пнд 19:58:49 59949649
>>599494
Блядь, да не говорю я про воздушный старт, я говорю про ракету, которая по форме сделана в виде самолета, чтобы когда топливо закончилось, она могла приземлиться самолетным способом, не нуждаясь при этом в полноценных крыльях. Или что, фалькон тоже воздушным стартом считать?
Аноним 07/09/20 Пнд 20:00:01 59949750
Стикер 63Кб, 250x250
250x250
>>599496
>Блядь, да не говорю я про воздушный старт
>>599482
>Почему бы не сделать первую ступень ракеты в виде самолета с несущим фюзеляжем? Взлетает, выкидывает вторую ступень, садится на аэродром downrange
Ты только что воздушный старт по определению.
Аноним 07/09/20 Пнд 20:02:31 59949851
>>599497
Ну окей, возьмем флакон. Взлетает, выкидывает вторую ступень, садится на баржу downrange. Флакон - воздушный старт?
Аноним 07/09/20 Пнд 20:06:58 59950152
>>599497
Кек, ты не понял, этот омич действительно не про воздушный старт.
Он хочет сажать первую ступень своей ракеты, но не никарсива, как у маска, а по-самолетному, и для этого делать её, натурально, в виде самолета.
Аноним 07/09/20 Пнд 20:08:25 59950453
>>599501
Не просто самолета, а самолета с несущим фюзеляжем, чтобы корпус ракеты заодно служил крылом, и не нужно было таскать полноценные крылья как на байкале.
Аноним 07/09/20 Пнд 20:10:15 59950554
>>599498
Нет.

>>599501
Но это воздушный старт, зачем есму самолет тогда?
Аноним 07/09/20 Пнд 20:10:30 59950655
>>599504
Это частности. А посадочный двигатель у этой фиговины отдельный будет?
Аноним 07/09/20 Пнд 20:11:39 59950756
>>599506
А нахуя ей посадочный двигатель? Нехай планирует как шаттл, S-маневры там всякие делает.
Аноним 07/09/20 Пнд 20:14:45 59950957
А, я кажется понял.
Надо такую же бесполезную штуку как шаттл, но еще хуже, т.к. большую часть будет лететь по пологой траектории и тормозить об атмосферу вместо нормального старта.
Генитально.
Аноним 07/09/20 Пнд 20:19:30 59951058
>>599509
>большую часть будет лететь по пологой траектории
Схуяли? Еще раз - это не самолет, это ракета в форме самолета. Запускаться будет именно как ракета, вертикально.
Аноним 07/09/20 Пнд 20:23:43 59951159
омсккосмос2020.jpg 84Кб, 1716x855
1716x855
>>599505
Говорю же, не понял. Но я телепат опытный, объясняю на картинке.
Аноним 07/09/20 Пнд 20:29:40 59951260
>>599504
Проблема в том, что мы в 2020 году и нам известно, что посадить ступень от ракеты вполне возможно - её не надо делать в виде самолета. Она вполне себе садится и просто в виде в виде ступени.
А ты предлагаешь другой режим полета навернуть что бы было красива.
Аноним 07/09/20 Пнд 20:31:03 59951361
>>599511
Ну да, все верно. Нужно затем, чтобы ступень было проще сажать. Для полного феншуя можно вторую ступень тоже сделать маленьким космопланом по типу дримхуйни или клипера. Получаем полностью многоразовую хуйню для полета на орбитальные станции (с грузом или огурцами). К Луне и Марсу огурцов катаем ядерным шаттлом, топливо для него возим на си дрэгоне. Все, я вам только что освоил всю солнечную систему.

>>599512
Посадить возможно, надежно посадить - нельзя.
Аноним 07/09/20 Пнд 20:44:21 59951562
>>599513
Ты хочешь скрестить ужа с ежом. Такие попытки предпринимались и раньше. У тебя получится хуевый самолет и посредственная ракета, которая должна будет тащить на себе всю конструкцию для самолетной посадки просто так.
Аноним 07/09/20 Пнд 20:47:58 59951763
>>599515
>У тебя получится хуевый самолет
А мне и не нужен охуенный самолет, от этого самолета требуется одно - сесть на ВПП.
>и посредственная ракета
Похуй на то, какая она посредственная, если она многоразовая.
Аноним 07/09/20 Пнд 20:52:42 59951964
>>599517
>мне и не нужен охуенный самолет
>Похуй на то, какая она посредственная
На такое денег дадут только в американской оборонке.
Аноним 07/09/20 Пнд 20:53:41 59952065
>>599482
Тебе отвечали уже вроде.
Ты только что Байкал-Ангара, или как там оно сейчас, Крыло-СВ, похуй.
Идея валидная, нечего добавить. Таких очевидных идей дохуя, просто никто не пробует из-за стагнации. А пока не попробуют, трудно сказать как оно будет на практике.

>>599512
>Проблема в том, что мы в 2020 году и нам известно, что посадить ступень от ракеты вполне возможно - её не надо делать в виде самолета.
Кому нам? Какой ещё 2020? Что посадить ступень технически возможно - было и ежу очевидно, это просто ракетодинамическая посадка. Вопрос был в количестве ПН, которое отжирает лишнее топливо, и в стоимости этого всего. Ступень, летающая назад в точку старта, теоретически может сэкономить включения (ресурс) движка, транспорт, и полезную массу при помощи ВРД. Но взамен требует дополнительной массы на ВРД, шасси и жесткость. Стоит ли того - это вопрос, вполне возможно что стоит, на бумаге цифры сходятся. Но может быть и нет, надо смотреть на практике. Может быть можно и вообще без ВРД обойтись, чистым планированием и гибридным возвратом (крутая траектория подъёма + импульс назад в апогее, тратим лишнее включение и немного горючки, но экономим массу на ВРД). Если б я был Илоном Маском, попробовал бы так.

>>599515
>Такие попытки предпринимались и раньше.
Какие попытки, ты поехавший штоле? Попытки во флайбэк бустер были?!
Аноним 07/09/20 Пнд 20:54:14 59952166
>>599515
>>599519
Это тред тупых вопросов, а не ответов.
Аноним 07/09/20 Пнд 20:57:36 59952267
>>599520
>Ты только что Байкал-Ангара
На байкале крылья, а я хочу несущий фюзеляж, ну там с мелкими крылышками для управления. Плюс байкал же боковой ускоритель, не?
Аноним 07/09/20 Пнд 21:04:13 59952368
>>599520
>шасси
Хотя чего это я, вообще и VTVL требует ноги, которые даже наверно тяжелей могут быть, т.к. амортизируют всю инерцию при избыточной вертикалке.

Ваще "если б я был Маском", я бы попробовал сделать флайбэк бустер так:
- избавился бы от ВРД нахуй, чисто чтоб попробовать выйдет ли
- пук в апогее назад, на полшишечки - остальную дистанцию до старта компенсируем планированием
- залил бы ВПП льдом или водой, или покрыл бы тефлоновой пленкой
- вместо шасси поставил бы фиксированные полозья (экономим массу и сложность)
- въебал бы ногу для фрикционного тормоза об полосу

>>599522
Т.е. отличие только в аэродинамической схеме? Так бы и объяснил, что тебе нужно что-то типа IXV. Так-то в байкале тоже несущий фюзеляж, сам посмотри на соотношение площадей. Крылья нужны лишь для улучшения аэродинамического качества, ну вот так у них цифры сходились. Можно ли без них обойтись - сказать невозможно, не реализовав. А гадать на кофейной гуще можно долго.

Боковой-то боковой, но на это похуй, это частности конкретного решения.
Аноним 07/09/20 Пнд 22:38:44 59954469
>>599511
Всё, теперь понял.
Плохо зделол, нималаца.
То что будет его тащить как лифтинг боди на посадке будет ебический драг создавать. Управляющие поверхности и механизацию в любом случае надо тащить, и из-за асимметрии планирующего полёта и необходимости по-хорошему симметрию на взлёте иметь, придётся делать изменяемые управляемые поверхности.

>>599515
>Ты хочешь скрестить ужа с ежом.
Двачую.
Байкал с драгом на старте.
Лучше уж ССТО а-ля венчурстар действительно.
Аноним 07/09/20 Пнд 22:42:32 59954570
>>599544
Нахуй не нужна симметрия на взлёте, и драг зависит от угла атаки.
>Лучше уж ССТО а-ля венчурстар
В альтернативной вселенной разве что
Аноним 07/09/20 Пнд 22:47:32 59954771
image.png 1248Кб, 704x944
704x944
>>599545
>Нахуй не нужна симметрия на взлёте
Компенсировать будешь ещё большим драгом из-за компенсирующих управляющих поверхностей тогда.
>и драг зависит от угла атаки.
У тебя лифтинг боди, у тебя йоба широкая, уже всё, ты драг получаешь не пуляя цилиндр.
>В альтернативной вселенной разве что
В этой. Ты его пускаешь, но зачем-то не довыводишь на орбиту.
Аноним 07/09/20 Пнд 22:53:30 59955172
>>599547
>зачем-то не довыводишь на орбиту
А нахуй он мне на орбите нужен? Развернуться, пукнуть и сгореть в атмосфере? А если мы делаем SSTO, да еще и с термозащитой, какая у него полезная нагрузка будет? Кубсат хоть влезет? Был ровно один вменяемый проект одноступенчатой ракеты, и то там вся суть была в том, что там была ракета из говна и палок для запуска на орбиту хуйни, которую не жалко проебать, вроде жратвы для огурцов.
Аноним 07/09/20 Пнд 22:54:17 59955273
>>599551
>А нахуй он мне на орбите нужен?
Вывести ПН.
Ясно всё с тобой, на конструктивный диалог не способен.
Пошёл на хуй.
Аноним 07/09/20 Пнд 22:56:16 59955374
>>599552
>Вывести ПН
А вторая ступень на что, блядь?
Аноним 07/09/20 Пнд 22:57:43 59955475
>>599547
Практически все ракеты в большинстве миссий летают на ненулевом АоА, внезапно.
>В этой.
ССТО по определению всегда хуже многоступенчатой схемы, лучше только в отсутствии одного разделения. Не надо вскрывать эту тему.
>У тебя лифтинг боди, у тебя йоба широкая
Не обязательно широкая. Вопрос в свитспоте подъёмная сила-температура-динамическое давление. Байкал тоже частично лифтинг боди. Но ващет я и говорю что некие крылья скорее всего нужны (я другой анон), узкая калоша с бодифлапами уровня IXV будет иметь слишком херовое качество для адекватного полёта и посадки. Наверно. А может и не наверно. С потолка не скажешь.

Потери естественно будут, что у байкалоподобной схемы, что у чистого несущего фюзеляжа. Причем больше будут не от поперечного сечения и не от лобового драга на ненулевом угле, а от волнового (резкие углы и отличие формы от Сирса-Хаака). Но "ебические" это громко сказано, вопрос что выгодней остаётся.
Аноним 07/09/20 Пнд 23:00:24 59955576
>>599554
>ССТО по определению всегда хуже многоступенчатой схемы
Только он предлагает недостатки ССТО и недостатки воздушного старта.
>Не обязательно широкая.
Лифтинг боди способная в планирование и посадку на впп? Широкая, драг обязателен.

Да говно у него идея, не надо внимания обращать уже.
Аноним 07/09/20 Пнд 23:02:38 59955677
>>599555
>Только он предлагает недостатки ССТО и недостатки воздушного старта.
Что блять? Где ты видишь у него ССТО и воздушный старт, наркоман? Он тоже наркоман, но даже мне ясно что он говорит про VTHL первую ступень.
Аноним 07/09/20 Пнд 23:05:28 59955878
>>599556
Блядь, ты тоже дебил, короче.
Пошли оба на хуй.
Аноним 07/09/20 Пнд 23:06:25 59955979
>>599555
>Только он предлагает недостатки ССТО и недостатки воздушного старта.
У тебя в голове разве что. Недостаток ССТО - это хуевое соотношение стартовой массы с полезной нагрузкой. Воздушный старт - это вообще не в тему, там задача затащить ракету повыше, чтобы сопротивление воздуха поменьше было и сразу высотный двигатель использовать можно.
Аноним 07/09/20 Пнд 23:07:13 59956080
>>599559
Иди на хуй уже и шизохуйню свою забери, это тупорылый бесполезный бред.
С дирижаблей запускай, еблан, пользы больше будет.
Аноним 07/09/20 Пнд 23:09:30 59956281
Аноним 07/09/20 Пнд 23:09:32 59956382
>>599560
Испанцы, кажется, собирались запускать пердузу со стратостата, не знаю, чем в итоге закончилось. Но ты реально долбоеб, у тебя понимание прочитанного нулевое, как ты вообще в обществе функционируешь.
Аноним 07/09/20 Пнд 23:13:08 59956783
>>599563
Долбоеб тут ты, засунь себе самолет в сраку и лети.
Флакон уже делает то что ты хочешь, только без бесполезного ебланского самолетства.

>>599562
Слился - обтекай, кал.

Пиздец, ебланы, вы башкой хоть чуть-чуть думайте перед тем как свою тупорылую хуйню топить за нее, я вас разбебал по фактам, бляди, а вы нихуя не угомонитесь и проджолжаетье за нее копротивляться, уебки, пиздуйте в Б и там свое говно предлагайте
Аноним 07/09/20 Пнд 23:17:10 59957084
Извините за скабрезности, вырвалось, целый день мозги ебали, а еще и вы тут свою хуйню несете, вот и сорвался.
Аноним 07/09/20 Пнд 23:18:13 59957185
>>599567
>я вас разбебал по фактам
Окстись, болезный, как ты нас мог разъебать по фактам, если ты даже не врубаешься в суть дискуссии, мудила? Тебе, блядь, два часа пришлось втолковывать суть простейшей системы, а ты продолжал что-то невнятно верещать из своего заткнутого хуем отчима верхнего ануса про какие-то ССТО и воздушные запуски.

>>599570
Поздно, я уже написал охуительный ответ, в котором разъебал твой разъеб как хачи дупло твоей мамаши.
Аноним 07/09/20 Пнд 23:19:39 59957286
>>599571
Иди на хуй, даун.
Твоя бредовая хуйня это хуйня. ФАКТ.
Все.
Иди нахуй теперь и не еби больше этим калом мозг, сука ебаная, блядь.
Тупое говно выдумал и носится как пятилетняя мразь, сука ебаная, завали ебальник и не пизди, хули ты мозги ебешь?!
Аноним 07/09/20 Пнд 23:21:12 59957387
>>599572
Как ты можешь судить о бредовости, если ты даже не понимаешь, о чем речь идет? И уж тем более не имеет права запрещать мне ковыряться в носу микромозг, предлагающий с земляшки летать на ССТО.
Аноним 08/09/20 Втр 03:22:20 59958188
На юпитере и сатурне воняет говной?

Чем воняет на уране и нептуне?
Аноним 08/09/20 Втр 08:33:05 59958789
>>599581
Сложно сказать чем именно там воняет, но воняет не очень круто.
На Уране и Нептуне тоже есть и аммиак и сероводород, так что воняет тем же, вероятно. Они находятся в определенных слоях, до верха не достаёт, до низа тоже.

Хуже всего несёт на Титане, там ты буквально сдохнешь от вони (цианистые соединения)
Аноним 08/09/20 Втр 11:36:06 59960490
>>599433
>чему там полыхать?
Энергия бывает не только химическая или ядерная. Есть еще другие виды энергии, включая еще неоткрытые. Например, если каждый нуклон чуть "квантово" колеблется сам по себе, выделяя крохотную энергию, то громадная куча этих нуклонов будет выделять достаточно энергии чтобы эта куча светилась аки звезда.
Аноним 08/09/20 Втр 11:57:12 59960791
>>599587
>Они находятся в определенных слоях, до верха не достаёт, до низа тоже
Щас. Если это газы или жидкости, их молекулы представлены во всем доступном обьеме, просто с разной концентрацией.
Аноним 08/09/20 Втр 13:23:24 59961792
1297012469629.png 27Кб, 311x234
311x234
>>599581
Там отсутствует нюхающий наблюдатель, поэтому нет смысла рассуждать о том, чем там воняет.
Аноним 08/09/20 Втр 13:27:27 59961893
>>599607
То-то я смотрю озон везде в земной атмосфере, а на Венере кислотный пар и до поверхности и до экзосферы достаёт. Нет, тонкостей дохуя.
Аноним 08/09/20 Втр 14:49:05 59962394
Стекло - аморфное твердое тело. А можно ли его запечь/залить/запилить так, чтобы после охлаждения стало кристаллическим телом? Каковы будут свойства?
Аноним 08/09/20 Втр 15:16:18 59962895
Аноним 08/09/20 Втр 15:22:38 59963096
>>599623
гугли кварцевое стекло и кристаллы кварца
Аноним 08/09/20 Втр 15:38:28 59963397
>>599617
Где то умер один физик-теоретик
Аноним 08/09/20 Втр 16:06:40 59963698
А фундаментальные частицы это все предел? Или и они тоже состоят из более мелкой фигни? Где дно то? А вдруг так и со вселенной?
Аноним 08/09/20 Втр 16:07:22 59963799
>>599636
>Где дно то?
Которое?
Аноним 08/09/20 Втр 16:08:00 599638100
>>599637
На которое опереться можно и считать за аксиому
Аноним 08/09/20 Втр 16:12:51 599642101
>>599636
Ну чет про преоны кукарекают, но пока нихера не ясно. FCC жди, в сто раз ярче БАКа, может чего нащупает.
Аноним 08/09/20 Втр 16:14:35 599644102
Аноним 08/09/20 Втр 16:16:38 599646103
Аноним 08/09/20 Втр 16:19:37 599647104
Аноним 08/09/20 Втр 16:26:05 599648105
>>599647
И че, очередную кишку под него копать будут?
Аноним 08/09/20 Втр 16:27:44 599649106
wy1cmey3q2n41.png 803Кб, 1080x1475
1080x1475
>>599647
2040. В 4 раза длиннее. Лять вангую будет найдено еще фундаментальнее фундаментальных частиц и в довесок ещё куча трейлеров и анонсов куда более фундаментальнейших фундаментальных фундаментальных частиц.
Где дно то ебаный в рот?! Кто спиздил дно?
Аноним 08/09/20 Втр 16:29:52 599650107
>>599649
Ох допрыгаемся, спалим видюху на компе, на котором вселенная симулируется.
Аноним 08/09/20 Втр 16:30:53 599651108
image.png 144Кб, 451x521
451x521
>>599648
Дык.
К тому моменту уже ДЕМО начнут испытывать, правда.

>>599649
Макс Планк его застолбил. 10е-35 метров днище.
Всего-то порядков двадцать добить осталось.
Аноним 08/09/20 Втр 16:31:47 599652109
>>599650
Не парься, она автоматом подмодели симулирует, потому ты и разрешается квантовое состояние в момент замера, а до этого просто более высокое моделирование идет.
Аноним 08/09/20 Втр 16:32:26 599653110
>>599651
>1 metre
>person
Карлан не человек же.
Аноним 08/09/20 Втр 16:34:33 599654111
image.png 1040Кб, 1300x1268
1300x1268
>>599653
По яйцам захотел, дылда?
Аноним 08/09/20 Втр 16:35:48 599655112
>>599647
А кто этот праздник жизни будет финансировать?
Аноним 08/09/20 Втр 16:37:08 599656113
image.png 52Кб, 350x350
350x350
>>599655
ЦЕРН, кто ж ещё. Точнее, её участники.
Аноним 08/09/20 Втр 17:17:12 599660114
>>599651
>Всего-то порядков двадцать добить осталось.
А там армяне в нарды играют.
>>599647
>Стоимость плана разработки оценивается в 21 миллиард долларов США
Лять, вот пустить всяких Игорьков Сеченых на тушонку, забрать деньги и строй мама нехочу по всей сибири эти ускорители. И стучи частицами как на пасху яйцами. Стой только и охраняй что бы местные обшивку на парники для огурцов не распиздили.
Аноним 08/09/20 Втр 17:18:56 599661115
>>599660
Какую обшивку, их под землёю строить надобно.
Догадаешься, почему, или пояснить?
Аноним 08/09/20 Втр 17:20:58 599662116
>>599661
Ты не до оцениваешь деревенскую смекалку, бутылку водки и штыковую лопату.
Аноним 08/09/20 Втр 17:22:52 599663117
Аноним 08/09/20 Втр 17:26:37 599665118
Аноним 08/09/20 Втр 17:27:20 599666119
Аноним 08/09/20 Втр 17:30:27 599667120
unnamed.jpg 41Кб, 512x384
512x384
Аноним 08/09/20 Втр 17:47:07 599671121
Аноним 08/09/20 Втр 17:50:50 599672122
А что определяет темную материю именно как материю?
Все что известно про темную энергию это то что она расширяет вселенную?
Аноним 08/09/20 Втр 17:54:49 599674123
>>599672
1. Гравитация.
2. Вроде да.
Аноним 08/09/20 Втр 20:14:02 599695124
Многоразовые грузовики на МКС когда?
Как-то жалко, что столько всего просто сгорает.
А были бы большие, можно было бы крупные эксперименты регулярно возвращать.
Аноним 08/09/20 Втр 22:28:26 599714125
Как ваще образуются планеты и приобретают свою гравитацию?
Аноним 08/09/20 Втр 22:30:05 599716126
Аноним 08/09/20 Втр 22:37:21 599718127
Звезды образуются из молекулярных облаков. А облака откуда берутся? Если они просто постоянно летают, то почему они все не схлопнулись в звезды еще 10 миллиардов лет назад?
Аноним 08/09/20 Втр 22:39:42 599720128
>>599718
Им много времени и случая на это надо. В молекулярных облаках постоянно звездообразование, и взрывы новых (и старых) звезд раскидывают их, провоцируя как новые звезды, так и разнося облака.
Аноним 08/09/20 Втр 22:42:05 599721129
>>599720
То есть, типа 100 солнечных масс скукожилось в звезды, а еще 1000 просто раскидало миниоблачками во все стороны? А потом, когда-нибудь, миниоблачка соберутся вместе и снова начнут скукоживаться?
Аноним 08/09/20 Втр 22:49:51 599722130
>>599721
Вроде того, да. Газиков полно летает повсюду, пока соберутся, пока допетрят кто куда скукоживаться будет, рядом кто пошустрее ебанет и разметает опять, а потом свежие газики с распидорашенной сверхновой таки соберут этих разгильдяев, да получится протозвезда и протопланетный диск с тяжелыми элементами.
Аноним 08/09/20 Втр 23:08:50 599731131
Объясните, пожалуйста, простыми словами, почему солнце так долго горит? Из того, что вычитал из статеек понял, что оно похоже на вечный двигатель, который теряет только чуточку своей энергии, но такие теряет. Мол, одно заменяет другое, затем соединяется и остаётся, но теряет 0,0001% своей мощи, который и образует излучение. Примерно так?
Аноним 08/09/20 Втр 23:12:30 599732132
>>599731
Солнце необратимо "сжигает" четыре миллиона тонн вещества каждую секунду, это переходит в излучение.
4е9 килограмм в секунду...
Только его масса 2е30 килограмм.
За миллиард лет оно "худеет" лишь на 3е25 килограмм.
1/10000 своей массы.
Аноним 08/09/20 Втр 23:13:11 599733133
>>599732
Одну стотысячную, нолик потерял.
Да даже и десятитысячная не такое значимое число получается.
Аноним 09/09/20 Срд 00:21:23 599752134
>>599733
Но ващет потухнет оно не из-за того, что масса закончится, а потому что тяжелее гелия оно ничего жечь не может: массы-то все еще будет дохуя, но она "не горит".
Аноним 09/09/20 Срд 00:23:00 599753135
>>599752
Разумеется. Я не говорил ничего про конец звезды, я про настоящее.
Аноним 09/09/20 Срд 00:29:39 599758136
>>599753
Ну, тогда еще надо упомянуть, что в процессе сжигания кроме потери 4 миллионов тонн массы, 600 миллионов тонн водорода превращается в гелий (завидуйте, алхимики!).
Аноним 09/09/20 Срд 00:33:39 599759137
>>599758
Само собой.
У меня много чего было, чего хотелось упомянуть, но решил не перегружать деталями, и просто цифры показать.
Аноним 09/09/20 Срд 10:26:08 599801138
Сап, спейсач
Расскажи, как ты понял, что оно тебе надо? Читать умные книжки, залипать в небо, копить с обедов на телескоп и иже с ним? Как имея чуть меньше, чем ничего, проникнуться космосом и начать сычевать ночью на улице, а не в интернете?
Аноним 09/09/20 Срд 13:28:13 599884139
159964667676264[...].png 14Кб, 580x381
580x381
159964686501275[...].png 50Кб, 440x519
440x519
Смотрел видос о смерти вселенной, там говорилось что все звезды догорят до желаза а потом умрут.
Но ведь у железа есть период полуроспада, оно на что-то распадется, что снова сможет поддерживать ядерные реакции. Или нет?

Алсо, поясните как работает эта термоядерная физика. Что за энергия связи как я понял, энергия что связывает однозврядные протоны вместе и почему ее меньше всего в железе? Почему ее вообще разное количество в разных изотопах?

Еще читал про термоядерные реакторы недавно. Почему дейтерий тритий делают не гелий с тремя нейтронами? Алсо, в ИТЭРе собираются юзать кинетическую энергию этих нейтронов. А куда деваются эти МэВ?
Аноним 09/09/20 Срд 13:32:53 599886140
Аноним 09/09/20 Срд 13:53:48 599902141
>>599884
>все звезды догорят до желаза
Не все, а только массивные. Солнце, например, сейчас жгет водород (как и все нормальные звезды), потом будет жечь гелий, а потом закончится.
>Но ведь у железа есть период полуроспада
Период полураспада есть у изотопов, а не у элементов. У железа есть стабильные изотопы, которые не распадаются.
>Алсо, поясните как работает эта термоядерная физика. Что за энергия связи как я понял, энергия что связывает однозврядные протоны вместе и почему ее меньше всего в железе? Почему ее вообще разное количество в разных изотопах?
На пальцах, чем больше нуклонов ("кусочков ядра") в ядре, тем больше связывающее их сильное ядерное взаимодействие. Но начиная с некоторых размеров ядра, электромагнитное отталкивание одинаково заряженных протонов становится все сильнее (электромагнитное взаимодействие слабее, зато не так быстро убывает с расстоянием). Но это очень примерно, конечно.
>Еще читал про термоядерные реакторы недавно. Почему дейтерий тритий делают не гелий с тремя нейтронами?
Гелий-5 нестабилен, распадается в гелий-4 с испусканием нейтрона.
>Алсо, в ИТЭРе собираются юзать кинетическую энергию этих нейтронов. А куда деваются эти МэВ?
Кинетическая энергия частиц - это температура, по сути. Так что никуда не деваются, разогревают вещество, а потом, по традиции, избыток температуры снимают охладителем и вращают им турбину, скажем. Короче, классический самовар.
Аноним 09/09/20 Срд 13:59:13 599905142
>>599884
>Еще читал про термоядерные реакторы недавно. Почему дейтерий тритий делают не гелий с тремя нейтронами?
В некотором смысле делают, но при слиянии в ядре образуется огромный избыток энергии, достаточный, чтобы тут же развалить его и заставить выплюнуть нейтрон. Плюс гелий-5 сам по себе нестабильный.

>Алсо, в ИТЭРе собираются юзать кинетическую энергию этих нейтронов. А куда деваются эти МэВ?
МэВ это и есть кинетическая энергия осколков. 14,1 МэВ у нейтрона, например, означает, что он выбрасывается с охуительной скоростью порядка 50,000 км/с, 1/6 скорости света, что не очень хорошо для окружающих материалов, в которые он врезается.
Аноним 09/09/20 Срд 14:09:40 599910143
15995718545980.png 289Кб, 451x1041
451x1041
Аноним 09/09/20 Срд 14:14:55 599912144
tim and eric mi[...].gif 4451Кб, 350x233
350x233
>>599910
.юуннелесвитна отч окьлот ыТ
Аноним 09/09/20 Срд 14:25:45 599917145
>>599912
?тен евзар яиртеммисрепуС
Аноним 09/09/20 Срд 14:28:35 599919146
>>599917
?олачан илИ !ценок отЭ
Аноним 09/09/20 Срд 14:33:01 599921147
supertheory of [...].jpg 15Кб, 300x225
300x225
>>599649
>Где дно то ебаный в рот?!
В ебиной теории всего. Вопросы останутся как минимум пока не подружат квантомех и гравитацию/относительность.
https://en.wikipedia.org/wiki/Theory_of_everything

А потом возникнут вопросы, как это всё применять. Ибо разобраться как оно работает - это одно. А научиться предсказывать что-либо практическое при помощи этого - совершенно другое. Так-то например и сейчас известно как просимулировать движение предмета в воде с произвольной точностью, вплоть до фундаментальных сил. Но практически это не реализуемо на сколько-нибудь серьезных масштабах, и не приближает тебя к решению проблемы турбулентности. В общем, фундаментальные частицы и физика высоких энергий - это ещё не вся физика.
Аноним 09/09/20 Срд 14:39:39 599924148
MV5BMTQ1ZmIzOTA[...].jpg 19Кб, 477x268
477x268
Аноним 09/09/20 Срд 14:45:23 599927149
5b742630b53b067[...].jpg 24Кб, 770x433
770x433
>>599919
Я вот все думаю сижу. А что если правда так и есть. И грань между нашим и "зеркальным" может выступать темная материя например.
>>599921
Но все же как связано между собой. Или атомам похуй че там вверху творится?
Аноним 09/09/20 Срд 15:24:41 599942150
Аноним 09/09/20 Срд 18:16:44 600012151
Детектировали ли какие-нибудь метеориты или прочие объекты падающие на Марс? Ну кроме того орбитера который наебнулся из-за использования даунских единиц измерения вместо метрических.
Аноним 09/09/20 Срд 18:28:44 600014152
askastro2.png 276Кб, 600x426
600x426
464800351025250[...].jpg 108Кб, 961x480
961x480
>>600012
Метеороид - то что летит в космосе
Метеор - то что летит и горит в атмосфере, именно что "падает"
Метеорит - то что уже упало и летит на поверхности.

Метеоры искали и Спирит/Оппортьюнити, и Кьюриосити. Гарантированно нашли вроде один только, это ж надо точно в кадр попасть. А метеоритов на поверхности лежит дохрена и больше.
Аноним 09/09/20 Срд 18:29:54 600015153
>>600014
А с орбиты не замечали? Вроде спутники для картографии и прочих исследований были
Аноним 09/09/20 Срд 18:31:53 600016154
>>600015
Спутник основной один, это MRO.
Только разницу до и после, но не сам метеор.
Аноним 10/09/20 Чтв 03:30:49 600094155
Что можно почувствовать на марсе, высунув задницу из скафандра?
Аноним 10/09/20 Чтв 04:24:15 600095156
>>600094
Процент геев-колонистов, набранных по квоте.
Аноним 10/09/20 Чтв 05:13:51 600097157
Аноним 10/09/20 Чтв 14:54:07 600125158
>>600016
Но ведь Марка Ватного отслеживали сразу несколькими.
Аноним 10/09/20 Чтв 15:24:15 600126159
Почему ничего не слышно про electric sail? Эстонцы года три назад запустили один неудачно (не раскрылся) и на этом всё стухло. А говорят, ебейшая технология, с которой можно бесплатно летать как в сторону Солнца, так и от него.
Аноним 10/09/20 Чтв 15:26:44 600127160
>>600126
Развивать надо, пока что раскрытие панелей нифига не надежно, а по тяге хуже ионных и плазменных движков. Китайцы вроде чего пилят, но у них развитие хуй знает как идет.
Аноним 10/09/20 Чтв 16:26:37 600132161
>>600126
Всё как и с подобными технологиями, надо отрабатывать, а ни у кого нет особого стимула пробовать что-то совсем новое. Подобных идей дофига, с тросами, парусами, надувнушками и чем только не.
Аноним 10/09/20 Чтв 16:50:44 600134162
>>600126
>Сила разгона корабля электрическим парусом в 200 раз меньше, чем у аналогичного по размерам солнечного паруса[8].
Вот в этом проблема. Из нитей надо строить настолько огромную залупу, что её уже солнечным ветром сносить начнёт. А тяги и так с гулькин хуй.
Аноним 10/09/20 Чтв 18:57:05 600147163
А какие батареи в космосе используют?
Обычные липолы же холод и перепады вряд ли будут держать?
Аноним 10/09/20 Чтв 19:17:04 600154164
>>600134
Насчет тяги не совсем так. Давление солнечного ветра в 200 раз меньше давления солнечного света, это правда. Но в электропарусе протону ветра необязательно сталкиваться с нитью, он импульс и пролетая рядом передаст через электростатическое поле, фотон так не может. То есть масса нитей запросто может получиться в 200 раз меньше массы пленки.
Аноним 10/09/20 Чтв 19:30:35 600156165
>>600147
Диды использовали серебряно-цинковые, в современных таки да, литий-ион. Само собой в терморегулируемом обьеме, на голом вакууме хз какая там будет температура вообще, от +120 до -100 может быть в зависимости от ориентации аппарата на Солнце и затенения Землей на низкой орбите.
Аноним 10/09/20 Чтв 19:32:55 600157166
>>600156
У лиона же циклы ограничены, это не проблема?
Аноним 10/09/20 Чтв 19:44:51 600158167
c2eb4991ed21cec[...].jpg 73Кб, 700x525
700x525
655493original[[...].gif 369Кб, 640x640
640x640
contentPIA18615[...].jpg 95Кб, 800x500
800x500
660303900[1].jpg 221Кб, 900x900
900x900
Люблю ТТУ спейсача. Есть вопрсоы:
1) В чём суть картинки на 4? Поясните. Типа, скоро юпитерная гравитация распидорасит орбиту Меркурия? И это при том, что Меркурий сильнее всего подвержено влиянию Солнца, не говоря о том, что там ещё влияния Венеры, Земли и Марса?
2) Прочитал "Аэлиту" Толстого. Не ожидал, что советская фантастика может зайти, но она зашла и даже очень. По книге появился вопрос - а где находятся Соацера и место посадки корабля-яйца Лося и Гусева? Я бы на карте Марса хотел увидеть точку, а то без этого не достаточно полная картина восприятия получается.
3) Почему Марс такой синий? На пикрилах куча синести. Что это? Откуда она?
4) Куда садятся все эти американские марсоходы? Как куда, на остров Дэвон! Что за место высадки и чем оно обусловленно? Почему, например, не высадиться на Олимп, в Маринер, в Элладу, на полярные шапки или в Лабиринт Ночи?
Аноним 10/09/20 Чтв 19:50:12 600160168
>>600158
1. Да.
https://www.newscientist.com/article/dn13757-solar-system-could-go-haywire-before-the-sun-dies/?ignored=irrelevant
Из-за прецессий орбит Ртуть может войти в резонанс с Питером и быть выпиздована на мороз.
2. Это в сайфач.
3. Небо синее.
4. Куда считают наиболее перспективным.
>Люблю ТТУ спейсача.
Единственный твердотопливный ускоритель тут - это мой пукан.
Аноним 10/09/20 Чтв 19:52:09 600162169
>>600160
>3. Небо синее.
Я про поверхность. Про то, что за материал, почему он такой синий и почему его там так много.
>Люблю ТТУ спейсача.
Я хотел написать ТТВ (тред тупых вопросов)
Аноним 10/09/20 Чтв 19:59:38 600163170
>>600162
enhanced-color photo. Оно на самом деле серое, но чтоб больше деталей бросалось в глаза, цвета преувеличивают.
Грубо говоря - другой баланс белого.
Аноним 10/09/20 Чтв 20:03:09 600164171
>>600163
Ну и нахуя? Сперва они Марс вырисовали своим цветокором до карсного как борщ, а теперь вот в синий выгнули. Нахуя, а главное зачем? Что, нельзя просто фотки без фотошопа выложить?
Аноним 10/09/20 Чтв 20:04:06 600165172
>>600164
Можно. Иди и смотри фотки без фотошопа. Кто виноват что ты на желтушной попсе их выискиваешь, вместо жпл, спейса или насы?
Аноним 10/09/20 Чтв 20:05:27 600167173
>>600157
Если держать все время заряженным, длительность работы сильно повышается. Телекоммуникационные спутники к тому же летают на высоких орбитах и затеняются редко. Много циклов будет у старлинков, но у них и ресурс маленький.
Аноним 10/09/20 Чтв 20:09:27 600168174
>>600167
Хотел про МРО спросить и его 15 лет на орбите, но сам нашел, что у него никель-водородные.
Аноним 10/09/20 Чтв 20:13:55 600169175
>>600165
Ну так дай ссылку, а то я-то рукожоп
Аноним 10/09/20 Чтв 20:14:27 600170176
>>600164
Это специально обработанные картинки, чтобы лучше было видно различие между камнями и разными типами грунта, никто и не пытается выдать их за то, что увидел бы человеческий глаз.
Аноним 10/09/20 Чтв 20:23:15 600172177
>>600169
Сейчас, только жопу тебе вытру.
Аноним 10/09/20 Чтв 20:34:22 600173178
image.png 923Кб, 1000x625
1000x625
>>600169
true color добавляй когда ищешь космические фотографии
Аноним 10/09/20 Чтв 21:05:20 600175179
clown.png 34Кб, 246x149
246x149
вы все пикрилы тут сидите
Аноним 11/09/20 Птн 02:32:54 600198180
PIA15289hires.jpg 283Кб, 1969x1024
1969x1024
xlbzmbr84m331.jpg 359Кб, 1607x1197
1607x1197
marsdial.png 146Кб, 850x442
850x442
>>600163
>>600164
>>600173
Это не спейсачерский метод. Берёте лезете в PDS насовский, тырите оттуда равки с MAHLI или какой хотите камеры, и калибруете сами. Или проявленные равки с сайта куриосити, их ещё проще калибровать, никаких цветопрофилей (но можно только точку белого поправить). На борту Куриосити прилетело несколько калибровочных мишенек, с монетой для MAHLI, Marsdial ака солнечные часы, и титановая для ChemCam'a. Последней вы воспользоваться сами не сможете (её лазером жгут и спектр измеряют, надо в архиве PDS брать готовые замеры), а вот MAHLI/Mastcam вполне. Делов-то в RawTherapee или фотошопе/лайтруме ткнуть в белую область пипеткой. Главное брать фотку мишени в наиболее близкое время суток, и из начала миссии, а то их уже давно пылью позаносило.
Аноним 11/09/20 Птн 02:39:42 600200181
>>600198
А оптика не запылилась?
Аноним 11/09/20 Птн 02:39:52 600201182
>>600164
Нельзя. Цвет это субъективное ощущение, а глаз - не спектрометр. Не то что двум разным людям, а даже тебе в разное время и в разных условиях один и тот же предмет может показаться окрашенным в разные цвета. Ты можешь даже сделать так, что красный тебе будет белым казаться, и это не какая-то иллюзия, а так и будет. В общем, и те фотки правильные, и эти. Вот например у анона >>600173 обе последние фотки - "как видит глаз".
Аноним 11/09/20 Птн 02:43:48 600202183
684581mainpia15[...].jpg 74Кб, 946x710
946x710
>>600200
Она прикрыта крышкой.
Аноним 11/09/20 Птн 02:44:53 600203184
1576MR008022001[...].jpg 98Кб, 640x768
640x768
>>600202
С откинутой крышкой.
Аноним 11/09/20 Птн 02:50:09 600204185
>>600198
https://www.sciencedirect.com/science/article/pii/S0019103520302529
А ещё пишут, что научились точно учитывать пылевой слой на мишенях. И что мишеньки лучше располагать вертикально на будущих аппаратах, т.к. с горизонтальных (даже вверх ногами) пыль слабо сносит.
Аноним 11/09/20 Птн 03:45:58 600206186
>>600168
У никель-водородных в три раза меньше удельная плотность энергии, чем у лития, и ОЧЕ высокий саморазряд. 20к циклов конечно хорошо, но и литий уже к 10к подбирается, особенно литий-титанат. А вообще никель-водород это тот же NiMH, только водород при зарядке не в металле растворяется, а в баллоне собирается. А еще они стоят на Хаббле (где проработали 19 лет до замены в 2009) и МКС.
Аноним 11/09/20 Птн 03:47:13 600207187
>>600206
Спасибо за это.
Можешь еще что интересное на эту тему рассказать?
Аноним 11/09/20 Птн 12:00:24 600227188
Поясните про КРИСТАЛЛ ВРЕМЕНИ простыми словами, а то я глянул вику и не понял.
Конденсат Бозе-Эйнштейна понял, кварк-глюонную плазму понял, супержидкость примерно понял, а вот КРИСТАЛЛ ВРЕМЕНИ ускользает от понимания моего быдломозга...
Аноним 11/09/20 Птн 13:20:38 600239189
15997513630600.png 751Кб, 669x945
669x945
Аноним 11/09/20 Птн 16:12:27 600256190
>>600239
Ученый изнасиловал журналиста.
Аноним 11/09/20 Птн 16:37:26 600260191
>>599482
Я рассказывал недавно, почему не сделали самолётную первую ступень.

Смысл любой ступени, в том числе первой, придать скорость следующей ступени.
Чтобы от этого был толк, самолёт этот должен разгоняться ну как минимум до 1.5-2к м/с, как обычная первая ступень.
А это около 5 махов, даже не сверхзвук, а пред-гиперзвук.

То есть, тебе нужен самолёт, способный разогнать тонн 50 полезной нагрузки до 5 махов.
Это очень сложно, НИОКР будет стоить десятки миллиардов.

И в итоге ты получаешь систему, могущую ну макс в 5-10 тонн на орбиту.

Тупо профит выхлопа стоить не будет, за эти деньги можно тысячу обычных ракет настругать.
Аноним 11/09/20 Птн 16:41:49 600261192
>>600260
Я слышал гиперзвуковой Air Force One в разработке.
Аноним 11/09/20 Птн 16:44:28 600262193
>>600261
Если скунсы займутся, то запилят как нехуй.
А вот если боенх какой, то не будет ничого.
Аноним 11/09/20 Птн 16:48:18 600264194
>>600262
Какие-то безымянные петухи Hermeus
Скунсы SR-72 делают, он вроде не большой
Аноним 11/09/20 Птн 16:51:30 600266195
>>600264
>двухлетний ноунейм стартап
>AF1
Проиграл. Ну у них это 95% попил, 5% пан-или-пропал идейные.

>SR-72
А вот тут не понел
>XXI
>разведовательные самолеты
У них до сих пор естьзадачи?
Аноним 11/09/20 Птн 16:55:39 600268196
>>600266
>Проиграл. Ну у них это 95% попил, 5% пан-или-пропал идейные
Я тоже так думал, зимой смотрел конференцию по аэронавтике и у них нихуя не было на тот момент, сейчас есть демонстратор двигателя и контракты с ввс
>У них до сих пор естьзадачи?
хуй знает, но вроде как полет запланирован на 2022 и они намекнули в трейлере топ ган самолет от них
Аноним 11/09/20 Птн 17:03:29 600269197
i(41).jpg 47Кб, 1440x1080
1440x1080
>>600266
>У них до сих пор естьзадачи?
Во даёт.
Аноним 11/09/20 Птн 17:31:33 600271198
>>600269
А, я все забываю, что там где они применяются нет зенитных систем достаточной силы.

Напомните, а что космического скунсы делали?
Аноним 11/09/20 Птн 17:34:48 600272199
image.png 4625Кб, 1650x1299
1650x1299
>>600271
Вроде только опытный аэроспайк, космос идет в основной локхид
Аноним 11/09/20 Птн 17:39:06 600273200
>>600272
>этот пик
Ух, охуенно, забыл что 71 для насы завербовали. Крутотень.

О, вот тупой вопрос еще вспомнился по организации. А как так получилось, что США стали невозбранно для своих ракет покупать российские двигатели? У них же это чуть ли не юридический зашквар.
Аноним 11/09/20 Птн 19:11:36 600275201
>>600273
Купили вместе со всей документаций. До того как не купили у них не было двигателей с закрытым циклом. Ещё они боялись что двигатели и инженеры уедут в Китай, Иран, Корею.
Аноним 11/09/20 Птн 19:12:44 600276202
Стикер 127Кб, 320x320
320x320
>>600275
>со всей документаций
НЕЕЕЕТ.
Реально!?
Аноним 11/09/20 Птн 19:14:46 600278203
>>600261
Ну во первых не "в разработке", а кампания, которая вроде как собирается пилить двигатель для него получила денег на поддержание штановна разработку. Причем прототипа двигателя, а не самолета. До гиперзвукового самолета от этого как до рака через Луну.
А в разработке для Эль Президенте пока только обычный сверхзвуковой самолет.

Тут еще вылезает такая мякотка - летать это одно, возить груз в десятки тонн это другое, а вот сбрасывать его на гиперзвуке это вообще третье. На SR-71 пытались реализовать запуск дрона на трех махах, и попытки закончились после того, как при втором запуске распидорасило самолет.
Аноним 11/09/20 Птн 19:16:31 600279204
>>600278
>кампания
Компания.
>А в разработке для Эль Президенте пока только обычный сверхзвуковой самолет.
Подробней?
> На SR-71 пытались реализовать запуск дрона на трех махах, и попытки закончились после того, как при втором запуске распидорасило самолет.
Я про это не слышал. ПН сверху была на фюзеляже?
Хорошо, что у них система аварийного спасения есть.
Аноним 11/09/20 Птн 19:27:35 600280205
>>600275
>До того как не купили у них не было двигателей с закрытым циклом
Поправлю - не было двигателей с закрытым циклом на керосине. Были на водороде.
>>600276
Да, они даже думали развернуть производство у себя, но как-то не срослось.
Аноним 11/09/20 Птн 19:29:17 600281206
lost all hope.jpg 79Кб, 766x960
766x960
>>600280
>когда я думал, что в роскосмосе не буду больше разочаровываться
Аноним 11/09/20 Птн 19:33:37 600282207
>>600281
Ну кто тебе враг, что ты узнаешь новости с задержкой в четверть века?
Аноним 11/09/20 Птн 19:43:03 600285208
LockheedM21-D21.jpg 270Кб, 1200x916
1200x916
Аноним 11/09/20 Птн 19:43:22 600286209
>>600280
>но как-то не срослось
По одной простой причине, двигатели из рашки были тупо вдвое дешевле, чем (в планах) локализованные американские.
Аноним 11/09/20 Птн 19:54:50 600290210
Аноним 11/09/20 Птн 20:12:27 600291211
>>600279
>А в разработке для Эль Президенте пока только обычный сверхзвуковой самолет.
>An Air Force One that flies at five times the speed of sound?
Аноним 11/09/20 Птн 21:25:58 600297212
>>600291
Ты не только заголовки читай.
Аноним 12/09/20 Суб 12:55:34 600331213
18fee08767d9d66[...].jpg 153Кб, 1280x720
1280x720
На Титане такая толстая атмосфера, и это не секрет. Но может ли она защитить от космической, солнечной и сатурнианской радиации?
Есть ли там ветра? Ведь по-идее на Земля ветра идут из-за малой прецессии (угла наклона оси, обеспечивающего смену времён года и сезонов) плюс смена дня и ночи, а так же наличие жидкости - воды - в большом количестве. На Титане есть тоже моря-окияны (хоть это и не вода), но там приливный захват и, насколько я знаю, нету никакого наклона оси.
Есть ли там магнитное поле, как у Ганимеда?
Аноним 12/09/20 Суб 17:11:19 600363214
15998885902830.mp4 17488Кб, 3838x1920, 00:01:21
3838x1920
Откуда это? И что это такое?
Аноним 12/09/20 Суб 19:59:37 600379215
seadragon.jpg 1020Кб, 4096x1532
4096x1532
II-D-6.jpg 1512Кб, 2544x3300
2544x3300
III-C-3.jpg 551Кб, 2544x3300
2544x3300
>>600370
Неправильно же!
Сопловый насадок маршевого двигателя второй ступени обёрнут вокруг баков первой.
Аноним 12/09/20 Суб 20:01:19 600380216
Аноним 12/09/20 Суб 20:01:39 600381217
>>600379
Нет, вообще нет. Откуда ты эти фейки нарыл?
Аноним 12/09/20 Суб 20:02:06 600382218
>>600363
Какой смысл делать видео в 4К, чтобы потом его так страшно зашакалить низким битрейтом?
Аноним 12/09/20 Суб 20:04:19 600383219
>>600363
Фейк, эти спецефекты за 10 баксов вообще позорище
Аноним 13/09/20 Вск 13:08:55 600448220
>>600363
>>600368
в менкайнде точно не было сидрэгона, он откуда?
Аноним 13/09/20 Вск 13:24:18 600449221
Аноним 13/09/20 Вск 13:26:19 600450222
И ни одна сука не написала годный сериал или нет.
Аноним 13/09/20 Вск 13:38:33 600451223
>>600449
Дурик, они там стол наклоняют вместе с актером и камерой. Он вначале упускает на секунду модельку и она вправо катится, еще без третьего колеса.
Аноним 13/09/20 Вск 13:46:33 600452224
Аноним 13/09/20 Вск 13:59:01 600455225
>>600450
Потому что это субьективно?
Мне сериал не понравился, потому что в нем очень большой "социальный аспект" - 90% фильма показывают нытье главных героев и их родственников, как все это сложно и круто, как плохо что геев не пускают в космос, а тян натуралов считают лесбиянками. а еще траву пропангандируют
Космоса очень мало и он больше служит фоном для истории "ни о чем". Так еще и сериал нереалистичный нихуя, во втором сезоне на луне будут летать шаттлы и бегать космодесантники с винтовками. в сериале так и не обьяснили, в чем смысл миллитаризации космоса

Но если ты тру спейсачер, то посмотреть стоит, ибо контента и так немного.

Аноним 13/09/20 Вск 19:41:16 600467226
>>600448
Был, в самом конце.
Аноним 13/09/20 Вск 22:24:03 600481227
Можно ли отправить килограмм говна в космос?
Вот существуют же услуги по отправке кубсатов стандартных размеров.
Можно сделать такой контейнер для говна квадратный.

Или откажут? Хотя в чем принципиальная разница, размер и вес соблюдён, деньги уплаченны...
Аноним 13/09/20 Вск 22:46:01 600487228
>>600481
Если нет риска, что твоё говно выплеснется из контейнера и заляпает ступень и другие спутники, то почти наверняка не откажут. Смотреть будут как на дебила (не без причины), но отказывать — вряд ли.
Аноним 13/09/20 Вск 23:14:04 600489229
>>600481
Почему откажут? Это же не клуб юннатов. У людей бизнес - возить всякое дерьмо в космос, и когда дерьмо действительно дерьмо - почему им отказывать, если ты платишь? Туда вон и прах покойников таскают, он от сушеного дерьма с инженерной точки зрения не отличается ничем.
Аноним 13/09/20 Вск 23:24:07 600490230
>>600481
Всем похуй, если к нагрузке нет претензий у регуляторов. Плоти бобло и соблюдай контракт. Операторы обычно ставят ограничения на кубсаты - нельзя ЖРД, нельзя хуйню под давлением, и т.п. Ну или льзя, но плоти ещё бобло за дизайн ревью своего говна, а его ты плотить обычно не готов, т.к. цена выйдет далеко за рамки задач кубсата
Аноним 14/09/20 Пнд 01:37:00 600498231
15582844337460.jpg 73Кб, 741x568
741x568
Почему гравиволны движутся со скоростью света? Разве гравитация - это не свойство пространства, и как известно, пространство не ограничено скоростью света, что позволило случиться инфляции?
Так пузырь альбукерки не надуть же, если искажать пространство быстрее скорости света нельзя.
Нипанимаю, памахити.
Аноним 14/09/20 Пнд 01:50:48 600499232
i(48).jpg 75Кб, 1014x906
1014x906
>>599618
>То-то я смотрю озон везде в земной атмосфере
Да.
Аноним 14/09/20 Пнд 01:54:08 600501233
>>600499
>концентрация
>Паскали
Что
Аноним 14/09/20 Пнд 02:10:20 600503234
i(49).jpg 77Кб, 1529x1079
1529x1079
Аноним 14/09/20 Пнд 02:41:44 600507235
>>600503
Смутило слово концентрация, ожидал проценты.
Но давление тут действительно более уместно.
Люблю бары вместо паскалей, это норма?
Аноним 14/09/20 Пнд 02:44:34 600508236
>>600507
>Люблю бары вместо паскалей, это норма?
Очевидно, что нужно мерять в psi.
Аноним 14/09/20 Пнд 02:45:56 600509237
атомный бугурт.jpg 24Кб, 314x292
314x292
>>600508
Затралено успешно.
Аноним 14/09/20 Пнд 02:47:13 600510238
>>600509
Это Незнаечка? Гаенайка?
Аноним 14/09/20 Пнд 02:53:26 600511239
https://arxiv.org/abs/1806.02751
Я правильно понел, что мелкую черную дыру 2MASS J05215658+4359220 нашли исключительно по шатанию компаньона, никогда не наблюдая никаких иных проявлений от самой ЧД?
Аноним 14/09/20 Пнд 02:55:09 600512240
>>600511
Многие экзопланеты так находят. И что?
Аноним 14/09/20 Пнд 02:58:08 600513241
>>600512
Планеты-то можно увидеть воочию помимо. Хабекс запустят, увидят.
ЧД так и останется неувиденной, или из-за того, что это система, на нее говно будет сыпаться, и нам просто не хватает разрешения разглядеть?
Аноним 14/09/20 Пнд 03:02:33 600514242
>>600513
Ну хуй знает.
>3.3+2.8−0.7 M⊙
10-20 км радиус, маловато. Если всерьез фонить не начнет, такую мелкую хуйню не разглядишь.
Аноним 14/09/20 Пнд 03:04:06 600515243
>>600514
Но в теории может же расшатать орбиту какого-нибудь местного юпидера и зохавав его, начать фонить своим диском?
Не, сам написал и понял, что такого не дождемся...
Аноним 14/09/20 Пнд 03:10:10 600516244
>>600515
Ну да, не дождемся. Вероятнее уж, что мы аццки прокачаем телескопы, наверное.
Аноним 14/09/20 Пнд 03:13:13 600517245
image.png 169Кб, 700x378
700x378
image.png 370Кб, 909x672
909x672
image.png 217Кб, 400x400
400x400
>>600516
Вот бы Лювоар-А запилили...
Аноним 14/09/20 Пнд 03:53:13 600519246
>>600513
>Хабекс запустят, увидят.
Как эта хуйня вообще собирается работать? Между телескопом и экраном десятки тысяч километров. Это можно один раз навести на одну звезду, но направление съёмки потом хуй изменишь.
Аноним 14/09/20 Пнд 03:55:05 600520247
>>600519
Так пока перепозиционируется коронограф, сам телескоп функционирует в режиме обычной обсерватории с пару недель.
Аноним 14/09/20 Пнд 04:31:03 600524248
>>600520
А коронограф будет на ТЯРД летать, чтобы дельты на over9000 перепозиционирований в течение срока службы хватило?
Аноним 14/09/20 Пнд 04:36:26 600525249
>>600524
Если ты никуда не спешишь и потихоньку гидразинчиком попукиваешь, то надолго хватит. Сренькнул и паришь до нужного места.
Не переживай за это, обсерватории пока что нет, а когда будет - будет выполнять основную миссию в сколько-то там экзопланетных систем, на что и расчитана.
ИРЛ не огурцач же где как получится выведешь, а потом на что останется наснимаешь.
Аноним 14/09/20 Пнд 08:50:01 600532250
>>599407 (OP)
Откуда взялись законы физики?
Аноним 14/09/20 Пнд 08:55:10 600533251
>>600532
Ньютон постановил. Будешь нарушать - пиздюлей вломит.

ЧТО БЫЛО ДО НЬЮТОНА, И ЧТО МЫ ПОТЕРЯЛИ:
Аноним 14/09/20 Пнд 10:36:06 600542252
Существует ли время в природе? Что такое время?
Аноним 14/09/20 Пнд 10:38:16 600543253
Аноним 14/09/20 Пнд 10:46:12 600544254
>>600543
А вот ебет тот факт, что по современным законам тело пролетающее В черную дыру. Для наблюдателя остановится на горизонте событий. Как?
Аноним 14/09/20 Пнд 11:54:54 600554255
>>600544
Ну так из-за горизонта ничего не поступает. Поэтому последнее что ты увидишь будет тело на горизонте.
Аноним 14/09/20 Пнд 11:55:23 600555256
Аноним 14/09/20 Пнд 14:00:22 600584257
>>600542
Да.
Время - мера течения процессов, пока мы не словили тепловую смерть и пока во вселенной есть что-то кроме фотонов, есть время.
Если останутся лишь фотоны - понятие времени потеряет смысл.
Аноним 14/09/20 Пнд 15:12:08 600601258
1600085527064.jpg 86Кб, 701x1024
701x1024
>>600498
Я вообще не понимаю, как большой взрыв мог произойти?
Вся материя была сжата в сингулярность — это литерали черная дыра, а, как известно, черные дыры не взрываются.
Аноним 14/09/20 Пнд 15:15:53 600602259
tim and eric mi[...].gif 4451Кб, 350x233
350x233
>>600601
Это те черные дыры, которые внутри горизонта событий и у которых сингулярность скрыта.
Выбирай более удобный для себя вариант:.
1. Это была не сингулярность.
2. Это была голая сингулярность без горизонта событий, отчего и распидорасилась.
3. Мы и так внутри горизонта событий.
Аноним 14/09/20 Пнд 15:16:15 600603260
>>600601
"Взорвалось" пространство, а не материя.
Аноним 14/09/20 Пнд 16:29:40 600617261
>>600603
Схуяли? Почему в черных дырах пространство не взрывается?
Аноним 14/09/20 Пнд 16:30:15 600618262
Стикер 255Кб, 500x500
500x500
>>600617
А с чего ты взял, что оно не взрывается?
Аноним 14/09/20 Пнд 16:32:43 600619263
>>600617
Возможно потому, что ЧД - часть пространства, а не целое.
Аноним 14/09/20 Пнд 22:24:14 600705264
1. Я ведь правильно понимаю, что нейтронные звезды тоже со временем остынут? Почему тогда термин "черный карлик" есть, а "черная нейтронная звезда" нет?

2. Сколько времени просуществуют звезды главной последовательности? Когда-то же для них закончится материал, но когда?
Аноним 14/09/20 Пнд 22:31:02 600711265
>>600705
>1. Я ведь правильно понимаю, что нейтронные звезды тоже со временем остынут?
Да.
>Почему тогда термин "черный карлик" есть, а "черная нейтронная звезда" нет?
Прост.

>2. Сколько времени просуществуют звезды главной последовательности?
Самые тяжелые около десятка миллионов лет всего. Солнечной массы - около десятка миллиардов. Самые легкие красные карланы - триллионы лет, насколько я помню.
Аноним 14/09/20 Пнд 22:45:50 600717266
Может быть в зоне убегания за галактическим диском крупная близкая галактика?
Аноним 14/09/20 Пнд 22:54:47 600721267
>>600717
Если бы была, её гравитационное влияние давно бы обнаружили.
Аноним 14/09/20 Пнд 23:05:05 600728268
>>600717
Даже две, Maffei 1 и 2.
Аноним 14/09/20 Пнд 23:16:24 600733269
Правильно ли я посчитал, что расстояние галактики треугольник до андромеды 700-800 тысяч св лет? И почему в таком случае не делается предположение, что прежде чем слиться с млечным путём, андромеда сольётся с треугольником?
Аноним 14/09/20 Пнд 23:37:38 600738270
>>600382
Чтобы обладатели калькуляторов наблюдали синий экран смерти из-за слабости видеокарты.
Аноним 15/09/20 Втр 01:48:34 600747271
>>600733
Ей пофиг на треугольник.
Аноним 15/09/20 Втр 02:14:59 600748272
>>600747
Они взаимодействуют в 12 раз сильнее, чем мы.
Аноним 15/09/20 Втр 02:37:01 600749273
>>600748
Да.
И при этом...
>>600747
>Ей пофиг на треугольник.
Аноним 15/09/20 Втр 10:48:47 600791274
>>600711
> Самые тяжелые около десятка миллионов лет всего.
Но ведь новые тяжелые звезды должны появится. Не может же быть такого, что через миллиард лет в галактике не останется тяжелых звезд.
Вопрос был в том, сколько еще просуществует вселенная в таком виде как сейчас - когда появляются новые звезды, вокруг них создаются планеты а на некоторых из них генерируется жизнь и тд.
Аноним 15/09/20 Втр 12:19:48 600803275
>>599407 (OP)

https://science.nasa.gov/biomarker-phosphine-discovered-atmosphere-venus

Как можно телескопом обнаружить МОЛЕКУЛЫ?

Не, я понимаю, что там какие-то спектры излучения специфические, хуё-моё но блядь,
смело говорить об открытии можно только физически "зачерпнув ковшиком" в пробирочку образец, не?
Аноним 15/09/20 Втр 12:42:31 600806276
>>600803
Спектры поглощения.
Аноним 15/09/20 Втр 13:09:42 600815277
Космос встал скален или нет?
Аноним 15/09/20 Втр 14:26:27 600837278
>>600791
А, ты в целом. Дохуя, звезды продолжают рождаться, да. Но вроде, в целом галактика становится тусклее, то есть звезд рождается все меньше и меньше. Но точных чисел я не помню. Вроде бы, снижение яркости порядка десятка процентов за миллиарды лет, но анус ставить не буду.

Но в любом случае, какие-то звезды еще триллионы лет будут: даже банальные красные карлики столько проживут, плюс хоть что-то да рождаться будет. Другое дело, что Вселенная, возможно, столько и близко не протянет, хотя стопудовой гарантии тебе тут никто не даст, это как бы гипотезы все пока.
Аноним 15/09/20 Втр 15:39:28 600871279
Там на Венере инопланетян нашли, а вы тут все вопросиками балуетесь.
Аноним 15/09/20 Втр 15:48:34 600877280
Бля а прикиньте что жизнь есть и на Венере, и на Европе и на куче других спутников с подповерхностным океаном. И только на марсе ее не будет, несмотря на весь форс вокруг него.
Блять, это же еще больше усилит парадокс ферми. Какой же будет ахуй, если окажется что на каждом втором камне во вселенной есть жизнь и только мы смогли эволюционировать.
Аноним 15/09/20 Втр 15:57:00 600881281
>>600803
Из-за квантовых эффектов, вещества поглощают преимущественно свет определенных длин волны. Причем это зависит не только от атомарного состава, но и от молекулярного (связанные атомы поглощают по-другому). Поэтому, посмотрев на спектр, можно примерно так почувствовать, что за вещества составляют атмосферу.
Аноним 15/09/20 Втр 16:03:26 600883282
>>600871
Меньше желтуху читай.
Аноним 15/09/20 Втр 16:52:02 600889283
>>600803
> Не, я понимаю, что там какие-то спектры излучения специфические, хуё-моё но блядь
> смело говорить об открытии можно только физически "зачерпнув ковшиком" в пробирочку образец, не?
Так зачерпнув ковшиком и будут спектры смотреть.
Аноним 15/09/20 Втр 17:03:40 600894284
image.png 100Кб, 300x305
300x305
>>600889
Когда у тебя физически исследуемое вещество, ты можешь масс-спектрометром прогнать, который ионизирует и "взвешивает" молекулы по времени пролета сопоставляя массу и заряд.
Аноним 15/09/20 Втр 17:47:56 600904285
>>600883
На сайте НАСА даже написали.
Брайденстайн высказался.
А ты "жёлтая пресса, жёлтая пресса".
Аноним 15/09/20 Втр 17:49:17 600905286
Аноним 15/09/20 Втр 17:51:33 600906287
>>600905
Это другое.
В этот раз британские мочёные.
Аноним 15/09/20 Втр 18:06:58 600909288
>>600906
А че за новость-то?
Вижу только про фосфин, PH3, это хуйня ничего не значащая же.
Аноним 15/09/20 Втр 18:21:57 600916289
>>600909
Это косвенный признак разумной жизни
Аноним 15/09/20 Втр 18:22:26 600917290
Аноним 15/09/20 Втр 18:35:32 600918291
>>600916
Вот это да, ставки повышаются. Уже не просто жизнь, а сразу разумная!
Аноним 15/09/20 Втр 18:58:50 600925292
original.jpg 231Кб, 734x718
734x718
>>599407 (OP)
На экваторе Луны гравитация понижена за счёт центробежной силы чи ни?
Насколько?
Аноним 15/09/20 Втр 19:09:25 600927293
>>600925
>На экваторе Луны гравитация понижена за счёт центробежной силы чи ни?
Да, чуть-чуть. Хотя правильнее так: гравитация немного компенсируется центробежной силой.
Аноним 15/09/20 Втр 19:15:52 600931294
>>600925
Радиус луны 1737.1 км
Скорость обращения 27.3 дней или 2.710-6/сек.
Ускорение a = rω2 = 1737100м х (2.7е-6/сек)2 = 12 663 459е-12м/сек2 = 1.266е-5 м/сек2.
Ускорение свободного падения 1.625 м/сек2, т.е. одна сотая процента этого ускорения нивелируется вращением.
Для сравнения расстояние до Земли 384,400 км, сила притяжения к земле на этом расстоянии составляет g = G
M/R^2 = 6.674×10−11 m3⋅kg−1⋅s−2 x 5.972 × 10^24 kg / (384 400 000 m)^2 = 0.002697м/сек2 = 2.697е-3м/сек2

Земля в сто раз сильнее воздействие оказывает, чем вращение луны.
Аноним 15/09/20 Втр 19:19:03 600932295
>>600931
Влияние Земли неправильно посчитал. Точнее, оно-то правильно, но по факту на точку на поверхности Луны будет влиять разность притяжения этой точки и центра масс Луны к Земле, то есть приливная сила. А она на порядки меньше.
Аноним 15/09/20 Втр 19:20:22 600933296
>>600932
А как правильно посчитать?
Аноним 15/09/20 Втр 19:26:29 600936297
>>600933
Как разницу же. Например, в точке, где Земля в зените, будет G/(R-r)^2 - G/R^2, где R - расстояние между Луной и Землей, а r - радиус Луны. Для надира будет G/(R+r)^2 - G/R^2. По факту там разность векторов и в обоих случаях она направлена от центра Луны. Для прочих точек все хуже (разность может быть направлена и к центру, и вообще куда-то "вбок"), но принцип тот же.
Аноним 15/09/20 Втр 19:27:33 600937298
>>600936
Я чет в режим хлебушка впал и плохо понимаю. Можешь вектора нарисовать?
Аноним 15/09/20 Втр 19:27:46 600938299
>>600936
На массу Земли не домножил только.
Аноним 15/09/20 Втр 19:29:25 600939300
>>600937
Погугли, я рисовать не умею.
Аноним 15/09/20 Втр 19:32:19 600944301
image.png 8Кб, 544x229
544x229
>>600939
Много ль надо, кружок, кружок, да чутка стрелочек.

Вот это имелось в виду?
Аноним 15/09/20 Втр 19:34:11 600945302
>>600815
Нет рагнорёк ещё не скоро. Пока расширяемся потихоньку. Приходи через пару триллионов лет.
Аноним 15/09/20 Втр 19:34:57 600946303
>>600944
Да, типа того. Только надо нарисовать вектор F из центра Земли еще, который больше F2, но меньше F1. И фактическая сила (относительно Земли) будет равна F1 - F для зенитной точки и F2 - F для надирной. А для прочих точек вектор Fx не параллелен F.
Аноним 15/09/20 Втр 20:03:40 600953304
image.png 427Кб, 414x533
414x533
image.png 958Кб, 532x962
532x962
Нафига столько пусковых площадок? Это же не аэропорт, в космос не запускают каждый день по 20 ракет.
Аноним 15/09/20 Втр 20:05:58 600954305
image.png 1482Кб, 1007x803
1007x803
image.png 475Кб, 566x488
566x488
image.png 196Кб, 353x266
353x266
Эбать, это еще не все.
Аноним 15/09/20 Втр 20:12:19 600955306
>>600953>>600954
То были шестидесятые, ракеты подготавливались долго и не всегда взлетали.
Аноним 15/09/20 Втр 20:46:22 600960307
>>600953
>>600954
Со времён космической гонки осталось, в основном. Зоопарк ракет - зоопарк стартовых комплексов.
Аноним 15/09/20 Втр 23:13:55 600987308
>>600931
>>600932
Так в итоге что больше - приливная сила или центробежная?
Аноним 16/09/20 Срд 01:13:51 600994309
>>600987
Приливное ускорение (вверх) в зенитной точке Луны: GM (1/(R-r)^2 - 1/R^2) = 6,67 10^-11 x 5,98 10^24 (1 / (3,85 10^8 - 1,74 10^6)^2 - 1 / (3,85 10^8)^2) = 2,45 10^-5 м/с^2.

Получается примерно в 2 раза больше, чем от центробежной силы, если я нигде не наврал.
Аноним 16/09/20 Срд 01:26:56 600995310
>>600994
Ну по порядку величины они совпадают. Любопытно.
Аноним 16/09/20 Срд 09:17:06 601022311
16001055109240.mp4 13670Кб, 550x400, 00:02:31
550x400
А можно ли использовать зеркальный телескоп как радиотелескоп?
Всего лишь меняя вторичную оптику в фокусе зеркала на радиооблучатель или излучатель для дальней космической связи
Зеркало так и так металическое напыление имеет отражающее, это металл, а так уж сложилось что сплошные металические слои отражают и радиоволны
А зеркала телескопов современных по 10 метров, такая апертура уже подходит для радио
Аноним 16/09/20 Срд 09:21:37 601024312
Аноним 16/09/20 Срд 09:29:19 601027313
>>601024
конструкция какая то уёбищная хуйня 54 метра радио, 2.5 оптика
я про полное использование апертуры зеркала для обоих диапазонов
Аноним 16/09/20 Срд 14:42:17 601067314
Немного не по теме но хз где еще спрашивать
>USAF’s digitally engineered aircraft to receive ‘e’ prefix, starting with Boeing eT-7A
В чем суть? Раньше на бумаге делалось что ли?
Аноним 16/09/20 Срд 15:01:38 601070315
Можете пояснить почему солнце считают желтым карликом, если на самом деле оно светит белым светом?
И как выглядят красные гиганты или оче горячие голубые звезды? Они тоже белые из космоса?
Аноним 16/09/20 Срд 15:23:33 601074316
planety.png 137Кб, 460x391
460x391
Что там с поиском 9. планеты?
Аноним 16/09/20 Срд 15:54:36 601077317
>>601074
Ее не существует.
Аноним 16/09/20 Срд 15:58:53 601078318
>>601070
Ну хую-то с двача явно виднее, как там на самом деле.
Аноним 16/09/20 Срд 16:15:09 601080319
well actually.jpg 55Кб, 400x400
400x400
>>601067
>Раньше на бумаге делалось что ли?
Ну да.
Аноним 16/09/20 Срд 16:17:12 601081320
image.png 44Кб, 1041x628
1041x628
>>601070
Про белый я поспорю, но тут интересный момент.
У Солнца же пик излучения приходится на УФ, почему его желтым прозвали?
Аноним 16/09/20 Срд 16:21:49 601082321
>>601070
Ну ты даёшь. Сам-то посмотри на Солнце.
Аноним 16/09/20 Срд 16:23:51 601083322
image.png 1101Кб, 1522x1151
1522x1151
>>601082
>посмотри на Солнце
У нас тред глупых вопросов, а не вредных советов.
Аноним 16/09/20 Срд 16:34:41 601085323
>>601070
Солнце светит зеленым светом на самом деле
Аноним 16/09/20 Срд 16:35:26 601086324
>>601080
Не, ну не в смысле раньше в 60ые, а в нулевые и десятые
Аноним 16/09/20 Срд 16:37:02 601087325
>>601086
CAD, но не без бумаг.
И вот где-то в 00/10 почались проекты целиком без бумаги.
Аноним 16/09/20 Срд 16:39:32 601088326
ece371c009a3438[...].jpg 54Кб, 1024x768
1024x768
САТУРН
Короче, есть одно солнце и вокруг него наворчивают круги Земля и Сатурн. При этом, в зависимости от их взаимного расположения, кольца Сатурна чуть-чуть наклоняются относительно наблюдателя на Земле.
А поворачиваются ли относительного самого Сатурна?
Просто я тут думаю, что кольца Сатурна - это не просто гигантская абразивная болгарка из пыли вокруг Сатурна, но ещё и поверхность отражения, а значит, систему "Сатурн - кольца Сатурна" можно использовать для отслеживания звёзд или даже какой-то одной конкретно взятой звезды (если повезёт). Используем его как прокси, или скорее как усилок, и будем херачить сигналы и сканировать им космос.
Аноним 16/09/20 Срд 16:40:10 601089327
>>601087
>И вот где-то в 00/10 почались проекты целиком без бумаги.
Так а почему только сейчас объявили об этом как будто еще вчера на бумаге все делали?
Или суть в том что все проекты ранее начинали в древние времена когда бумага была нормой, а этот и последующие целиком в цифре?
В любом случае, зачем префикс?
Аноним 16/09/20 Срд 16:40:48 601090328
>>601088
Он не усилок, он наоборот поглотит сигнал, лучше напрямую сканировать.
Аноним 16/09/20 Срд 16:41:17 601091329
>>601089
Суть в том, что они тормоза и медленно принимают всякие такие решения.
Аноним 16/09/20 Срд 16:44:35 601092330
parsolar-radiat[...].jpg 54Кб, 460x303
460x303
>>601081
Во-первых, на синий, а не на УФ, а во-вторых, хули ты in space принёс? Ты в вакууме живешь?

Спектральные классы были созданы задолго от космических телескопов, и основаны на наземных наблюдениях.
Аноним 16/09/20 Срд 16:46:11 601093331
>>601081
>>601082
Так ему атмосфера мешает, делая его лучи желтыми. А на закате из-за этого же эффекта оно краснеет.
Но в космосе оно выглядит белым, а не желтым.
Аноним 16/09/20 Срд 16:46:41 601095332
>>601093
>на уровне моря все равно синий а не желтый
Аноним 16/09/20 Срд 16:53:37 601099333
>>600945
Да вы охуели, мне тут с начала тысячелетия судный день обещают и нихуя не происходит.
Аноним 16/09/20 Срд 16:53:55 601100334
Eyesensitivity.png 99Кб, 669x587
669x587
>>601095
У тебя в голове глаза стоят, а не спектроскопы, у них другая кривая чувствительности. Я даже хуй знает, ты Солнце вообще давно видел? Где ты там синий разглядел?
Аноним 16/09/20 Срд 16:56:16 601101335
>>601100
Я тебе про спектр говорю. Не собираюсь своими глазами жертвовать чтобы смотреть на ослепляюще белый шар плазмы и лишаться зрения, для этого есть приборы и эти приборы показывают что у солнца пик в синем цвете.
Аноним 16/09/20 Срд 16:56:42 601102336
>>601070
По цветовой температуре.
Аноним 16/09/20 Срд 16:56:59 601103337
>>601100
А почему вода синяя?
Аноним 16/09/20 Срд 16:59:55 601104338
>>601103
Поглощает красный больше, отражает синий больше.
Аноним 16/09/20 Срд 17:09:57 601107339
>>601090
>Он не усилок, он наоборот поглотит сигнал
Чому? Площадь отражающей рабочей поверхности колец больше аналогичного у самого Сатурна
Аноним 16/09/20 Срд 17:11:42 601109340
>>601107
Они не усилят сигнал, они не направят его, это бессмысленно
Аноним 16/09/20 Срд 17:13:35 601110341
PlanckianLocus.png 460Кб, 1300x1462
1300x1462
>>601101
А я тебе говорю, что спектральные классы придуманы больше сотни лет назад и соответствуют тому, какой примерно цвет видел бы человек, если бы ему в глаза посветили таким спектром. "Пик" вообще нихуя не значит, важна именно эффективная температура.
Аноним 16/09/20 Срд 17:16:39 601113342
>>601110
Человеческие глаза говно, настроенное выискивать тигров в траве, они не прибор и не видят настоящие спектры. Смотреть надо на спектроскоп и он показывает какой цвет? Правильно, синий.
Аноним 16/09/20 Срд 17:30:00 601114343
>>601081
Потому что глаз человека не видит УФ. Очевидно, не?
Аноним 16/09/20 Срд 17:31:14 601115344
Аноним 16/09/20 Срд 17:32:35 601116345
>>601109
рептилоид, плиз, вы же за нами именно так и следите. а сатрун с кольцами - ваше всевидящее око. почему стрелочку обратно не повернуть?
Аноним 16/09/20 Срд 17:32:57 601117346
>>601114
Да не корми ты этого долбоёба, все с ним и так уже ясно.
Аноним 16/09/20 Срд 17:35:06 601119347
>>601116
Не тупи, плоский диск просто поглотит дофига радиации и отразит примерно поровну, ни о какому силении нет речи, с чего ты придумал усиление?
Аноним 16/09/20 Срд 17:36:00 601120348
>>601117
Это не я долбоеб, это вы долбоебы, я прав, на моей стороне объективная истина, дохуя картинок подтверждают реальность, что пик солнца не желтый нихуя. ФАКТ.
Нахуя вы копротивляетесь за пиздоболию? Промытки дохуя, мозга нет чтобы признать реальность?
Аноним 16/09/20 Срд 21:26:18 601194349
>>601100
>У тебя в голове глаза стоят, а не спектроскопы, у них другая кривая чувствительности.
Другая чем что? Глаза не спектроскопы вовсе не потому, что у них кривая чувствительности какая-то не такая. А потому, что там три компоненты всего, три скалярных значения на точку, а не полный спектр на точку (одномерная матрица = вектор). Поэтому два разных спектра могут давать неотличимый цвет, а объекты одного цвета под одинаково выглядящим (но имеющим разный спектр!) овещением могут казаться разными. https://ru.wikipedia.org/wiki/Метамерия_(цвет) С RGB-камерами то же самое.

>>601070
> Можете пояснить почему солнце считают желтым карликом, если на самом деле оно светит белым светом?
Классификацию звезд придумали на основе классических линий спектра.
"Белого света" же не существует. Цвет это субъективное ощущение в мозгу, а белый это точка, относительно которой ощущаются оттенки (да, это и в мозгу так же работает, не только у вас в лайтруме для АЧТ-спектра). Нехитрым экспериментом ты можешь обмануть глаз так, что зеленая скатерть будет казаться тебе белой, а всё остальное вокруг будет смещено в другие цвета относительно неё. На короткий срок, потом мозг адаптируется опять.
Аноним 16/09/20 Срд 21:51:53 601205350
15522343060950.jpg 56Кб, 800x529
800x529
Поясните за нейтрино. Википедия утверждает, что оно участвует в слабом и гравитационном взаимодействиях. Но словить нейтрино детектором тот ещё квест, и ловятся они туда только на высоких скоростях и то редко.

Я правильно понимаю, что нейтрино низких энергий ловятся ещё хуже и вообще хуй проссышь, сколько их там мимо пролетает? Что если вот эта вся тёмная материя, которую вообще никак не сдетектить (кроме как по гравитации), и есть эти самые толпы нейтрино, давным-давно испущенные звёздами в протон-протонных реакциях, и за давностью лет растерявшие скорость и энергию?

Просьба обоссать фактами данную гипотезу.
Аноним 16/09/20 Срд 21:54:56 601206351
>>601205
Наверное, но те типы нейтрино с которыми мы работали не поясняют такую массу.
Аноним 17/09/20 Чтв 00:59:49 601229352
Почему есть приливные захваты и почему они именно такие?
Представим планету-луну на орбите б0льшей планеты. Ближняя половина приливно захвачена, "стекаясь" весом в точку между материальной точкой ядра планеты-луны и планеты-гиганта. С противоположной от материальной точки ядра планеты-луны находится ещё половина планеты, которая не держится захватом так же, как и ближняя половина. И рассматривая эти половины отдельно, мы неотвратимо получаем разность моментов инерции и, следовательно, вращения.
Получается, дальняя половина планеты всегда будет вырываться и набигать по инерции в направлении движения луны, в то время как ближняя половина - в этом плане всегда бьудет притормаживать. А так как эти две половины замкнуты общим телом, то стремление дальнее половины перенаправится вниз, на нижнюю половину в набигающем направлении.
И это значит, что приливнозахваченные планеты обязаны всё равно вращаться.
В чём ошибка в суждениях?
Аноним 17/09/20 Чтв 01:12:56 601230353
>>601229
Энергия вращения захватываемой планеты тратится на её деформацию.
Аноним 17/09/20 Чтв 11:34:11 601275354
https://www.youtube.com/watch?v=ShC63MiURrc&list=PLIIOUpOge0LtW77TNvgrWWu5OC3EOwqxQ
Правильно ли я понимаю, что сделав такую гигантскую сферу дайсона из солнц, можно жить в черной дыре, за горизонтом событий? Значит, черные дыры особенно сверхмассивные это не только смертельно опасные штуки?
Ух бля, поместить звезды со всего ближайшего скопления галактик в радиус пары световых лет и лампово жить в созданой чд до конца своих дней.
Аноним 17/09/20 Чтв 12:07:37 601278355
>>601277
> которые даже с планеты выбраться не могут и на Луну 50 лет не летали, вы о чем вообще.
А нахуя? Чтобы доказать тебе, что человечество может? Если честно, я не вижу смысла даже в артемиде. Просто мега попил бюджета уровня аполлона, который существует хуй знает зачем. Но если при аполлоне люди узнали о множестве новых технологий, насса благодаря ему создалась практически с нуля, то артемида с ее sls - просто переиспользование движков 40 летней давности
И я просто задал вопрос про чд, а не пытался начать обсуждение о футуризме.
Аноним 17/09/20 Чтв 13:36:26 601292356
>>601284
Раз такой умный, то скажи, как этот "шаг" продвинет человечество?
Максимум - проспонсирует несколько частных компаний. Но они и без артемиды ноомально живут, особенно спейсх.
Аноним 17/09/20 Чтв 15:56:36 601331357
Образование против СТО. Приведем отзывы В.И. Секерина в его книге [11] по практике преподавания в школах и вузах теории относительности. «Теория относительности формировалась постепенно, большую подготовительную работу проделали ученые Э. Мах, А. Пуанкаре, Г. Лоренц и другие, но у них был свой взгляд на теорию относительности, отличающийся от позиции Эйнштейна. За время существования теории относительности, в понимании природы электромагнитного излучения наука не продвинулась вперед. Сформированная релятивизмом методика познания, в котором математические обозначения и графические символы принимаются за реальные объекты и изучаются, ведет в тупик. В настоящее время теория относительности является тормозом в мировой науке. Теория относительности, как и всякое проявление философского идеализма, особо пагубное влияние оказывает на неокрепшее сознание юношества, так как ее идеи нельзя понять, нельзя соотнести, согласовать, уложить в систему с ранее полученными знаниями, их можно только принять на веру и запомнить. Поэтому преподавание теории в школах и вузах ведет к воспитанию комплекса неполноценности, когда, приложив максимум усилий, человек ничего не понимает и считает причиной этого свои способности, либо двурушничество, когда при непонимании, утверждается вслух, что все понятно. И во всех случаях воспитываются идеологическая всеядность, эклектизм и отсутствие убеждений
Аноним 17/09/20 Чтв 15:59:22 601332358
>>601277
> что если выкопать пещеру на Солнце и жить внутри

Солнце это гигантский углеродный шар, который извергает водород в окружающее пространство, температура поверхности на уровне 100 градусов цельсия и нище. Это почти абсолютно чёрное тело. Свет = ионизация разогретого водорода.
Аноним 17/09/20 Чтв 16:01:23 601333359
>>600925
Радиус гравитации Луны всего 60 000 км, делай выводы
Аноним 17/09/20 Чтв 16:03:38 601334360
>>601333
>Радиус гравитации Луны всего 60 000 км
А дальше она отключается?
Аноним 17/09/20 Чтв 16:11:54 601335361
>>601292
Лезь обратно в пещеру
Аноним 17/09/20 Чтв 16:13:58 601336362
>>601334
Ну да, в ксп не играл чтоли?
Аноним 17/09/20 Чтв 16:39:38 601342363
>>600925
Гравитация одна и та же (ускорение свободного падения, точнее). Немного понижена, но за счет несферичности Луны (сжата вдоль экватора), как результат на экваторе ты находишься чуть дальше от центра. При этом разница намного меньше, чем естественные вариации гравитации Луны от неравномерного распределения её массы.

А ниже за счет центробежной силы - сила тяжести.
Аноним 17/09/20 Чтв 16:40:08 601343364
fukken lold spc.jpg 161Кб, 640x400
640x400
>>601332
>Солнце это гигантский углеродный шар
Аноним 17/09/20 Чтв 16:40:32 601344365
>>601278
>Чтобы доказать тебе, что человечество может?
Да, было бы охуенно.
мимо
Аноним 17/09/20 Чтв 16:43:03 601345366
>>601342
Хотя что-то я прогнал конечно, ж тоже ниже. Можете пиздить ногами.
Аноним 17/09/20 Чтв 17:17:11 601351367
>>601334
>>601343

Быстрая проверка на дилетанта

Кто такой Хевисайд? Что открыл?
Аноним 17/09/20 Чтв 17:18:43 601352368
>>601351
Открыл лунный кратер, а что?
Аноним 17/09/20 Чтв 17:22:59 601353369
>>601351
Это робот в фолаче 4, который на деревянном корабле стоит. Открыл огонь из корабельных орудий по набигающим рейдерам.
Аноним 17/09/20 Чтв 17:33:13 601356370
154799718116621[...].jpg 45Кб, 495x268
495x268
>>601355
Твой пост напомнил эту копипасту:

Блять больше всего я ненавижу мудаков которые говорят , что космос не нужен.
Я всегда ссу на ебало этим уродам, которых к сожалению огромное количество. Сраные ебаные кретины, тупорылое быдло мечтающее о теплом угле, вкусной еде и мясной дырке. Неужели сложно понять простую вещь - Движение вперед это охуенно. Фронтир, неизведанное, полететь на Марс потому что можем и дальше.
И вот эти чмыри которые ненавидят космос, видят только унылое его использование приходят в спейсач и начинают кукарекать, что Марс не нужен.
Сдохните твари. Все что создано вокруг вас руками человека создано наперекор таким как вы. Все время существования человечества мы волоком тащим вас визжащих,упирающихся еблом по говну, камням и грязи вперед к прогрессу и таки протащили из обезьяны к запускающим в космос корабли разумным существам.
Аноним 17/09/20 Чтв 17:40:39 601357371
>>601355
Большинство будет только визжать, что снято в голливуде.
Аноним 17/09/20 Чтв 17:41:51 601359372
>>601357
Наверное, не большинство, а громкое пиздящее меньшинство, которое лишь кажется большинством из-за того, насколько громко оно визжит.
Таких надо замолкать, они вредят человечеству.
Аноним 17/09/20 Чтв 17:55:52 601362373
>>601359
А потом замолчат тебя за троцкизм
Аноним 17/09/20 Чтв 17:56:52 601364374
>>601362
Меня-то за что? Я ратую за добро и пользу для человечества.
Аноним 17/09/20 Чтв 18:06:53 601368375
image.png 157Кб, 563x316
563x316
>>601367
Эхо-комнаты в пейсбуках и утубах позволяют дегенератам отрицать такую объективную реальность как польза вакцинации, форма земли или даже ебаные маски для лица.
Они могут купаться в своей тупости и безнаказанно заражать ей (и не только) остальных.
Аноним 17/09/20 Чтв 18:08:06 601369376
>>601352
>>601353
Вывод: вы не имеете к науке никакого отношения, а следовательно не смейте тут отвечать на вопросы, только людей запутаете всякими глупостями навроде пространства времени.
Аноним 17/09/20 Чтв 18:09:43 601370377
>>601369
Это ты не имеешь отношения и права отвечать на вопросы.
Нормальный адекватный умный человек бы просто написал по фактам что к чему, а не пытался всякими ноунеймами подъебывать других, чмоха ебаная. Пошёл в пизду, дегрод, и больше тут ничего не пиши, мразота.
Аноним 17/09/20 Чтв 18:16:09 601371378
>>601333
>Радиус гравитации Луны всего 60 000 км
А приливы-то и не знали.
Аноним 17/09/20 Чтв 18:17:02 601372379
>>601371
Как флаттарды объясняют приливы, кстати?
Аноним 17/09/20 Чтв 18:30:59 601376380
>>601373
>не будет ничего чтобы их как-то опровергнуть в космосе.
Для подтверждения формы земли в космос летать совершенно необязательно. Эратосфен успешно замерил размеры более 2000 лет назад.
Достаточно не быть идио-... Ох...

>>601374
А, точно, плоскодауны же еще и религиобляди.
Аноним 17/09/20 Чтв 20:52:23 601409381
Launch-Photo-2-[...].jpg 63Кб, 1024x576
1024x576
Почему за ракетой обычно не видно горячего следа?
Аноним 17/09/20 Чтв 20:58:13 601412382
>>601409
Обычно такого фона нет, на фоне неба не видно.
Поищи на фоне сооружений - сразу увидишь след, у всех ракет есть.
Аноним 17/09/20 Чтв 22:12:02 601427383
>>601373
>Амеры обосрались создав такой длинный промежуток между полётами на Луну, что никто не верит уже, но если бы слетали на Луну раньше еще раз допустим во времена буша, то было бы легче.
Хотя же можно лазером посветить на луну, который отразится.
Аноним 17/09/20 Чтв 22:19:31 601429384
>>601427
Думаешь, это доказательство для быдла?
"Я лазирнай укаской пасвитил и нифига нивидна, ты пиздиш всё!"
Аноним 17/09/20 Чтв 22:30:34 601431385
>>601429
>>601427
Плохой аргумент. Наличие уголкового отражателя доказывает только то что туда опустился какой-то аппарат, может быть совсем простейший. Не более того.

>>601372
https://fantlab.ru/work10670 Именно это не "объясняется", но на подобные вопросы отвечает. В целом забавное сочинение.
Аноним 18/09/20 Птн 01:06:22 601450386
Можно ли спастись от электромагнитного излучения, если делать постройку внутри клетки Фарадея?
Например, на поверхности безатмосферных планет или на орбите планет это бы могло снизить вредоносные влияния космоса.
А на Земле, наверное, спасло бы от даже от прямого удара молнией
Аноним 18/09/20 Птн 01:07:21 601451387
>>601450
>А на Земле, наверное, спасло бы от даже от прямого удара молнией
Так да.
Электрики в металлизированных костюмах работают, чтоб ток по ним шел.
Аноним 18/09/20 Птн 01:11:32 601453388
Знаю, не совсем космос, но в космосе тоже актуально.
Как бесстыковые рельсы ведут себя при разных температурах? Они же расширяются/сужаются от температуры, и это тонкие рельсы.
Какая-нибудь рельсовая ускоритель будет менять свои размеры если на луне построить.
Аноним 18/09/20 Птн 01:28:41 601454389
>>601450
>Можно ли спастись от электромагнитного излучения, если делать постройку внутри клетки Фарадея?
Пока за насыщение не выйдешь. Т.е. от прямого попадания эми-боеприпаса не спасёт.
Аноним 18/09/20 Птн 01:41:14 601456390
>>601454
>Пока за насыщение не выйдешь
Что за нысщение? Как его рассчитать? Что будет при выходе за насыщение?
Будет ли ловить телефон или вайфай в клетке Фарадея?
Аноним 18/09/20 Птн 01:41:51 601457391
Стикер 127Кб, 512x412
512x412
>>601456
>Будет ли ловить телефон или вайфай в клетке Фарадея?
Аноним 18/09/20 Птн 03:05:45 601461392
>>601456
>Что за нысщение?
Гугли нахой. https://www.booksite.ru/fulltext/1/001/008/072/370.htm
>Как его рассчитать?
По таблицам гистерезиса.
>Что будет при выходе за насыщение?
Поле намагниченного ферромагнетика будет меньше внешнего поля. Силовые линии будут простреливать клетку фарадея, и тоби пизда.
>Будет ли ловить телефон или вайфай в клетке Фарадея?
Не будет, там сигнал по мощности с гулькин хуй. Но длина волны маленькая, поэтому сетка нужна мелкая, иначе волна в дырку пролезет.
Аноним 18/09/20 Птн 03:18:33 601463393
>>601456
>Будет ли ловить телефон или вайфай в клетке Фарадея?
В незаземленной будет, за счет переизлучения самой клеткой. В заземленной зависит от конструкции заземления - проводники на некоторых частотах могут быть изоляторами, это широко используется в конструкциях антенн.
Аноним 18/09/20 Птн 10:53:10 601494394
Так ли все плохо у РФ, как смеются в интернете? Наша космонавтика на дне что-ли? Видел статистику, что китайцы делают больше пусков, чем мы. Для меня это вообще шок. Когда китайцы вообще успели в космос? правда я так же слышал, что у них там пиздец и каждый второй пуск трупы
Аноним 18/09/20 Птн 11:52:31 601503395
>>601461
А если сетка будет мелкая, как плетение на ткани, сможет ли она контрить УФ? А как избежать насыщения? Можно ли сделать клетку Фарадея с выводом, например, на лампочку, которая бы светилась тем интенсивнее, чем больше жрала бы поля клетка? Таким образом и клетка разряжается и уровень яркости лампы показывает, насколько снаружи опасно
Аноним 18/09/20 Птн 11:57:44 601506396
>>601494
>спойлер
Всё так, а у илона маска вообще до орбиты живыми только один раз долететь получилось, так что мы всё ещё впереди.
Аноним 18/09/20 Птн 13:48:52 601523397
hazanov shtou.jpg 26Кб, 339x255
339x255
>>601494
>каждый второй пуск трупы
Аноним 18/09/20 Птн 16:25:05 601543398
>>601453
>в космосе тоже актуально
>Как бесстыковые рельсы ведут себя при разных температурах?
Космический лифт хуйня, теперь электричка на орбиту модно?
Аноним 18/09/20 Птн 16:32:19 601544399
Поясните насчет "наблюдателя" в квантмехе. Правильно я понимаю, что это не дядька в лабораторном халате, а любое взаимодействие с другой частицей?
Аноним 18/09/20 Птн 17:08:21 601545400
>>601543
Кстати, а как лифт должен держаться за трос? Тем более что трос утолщается кверху.
Аноним 18/09/20 Птн 17:24:43 601546401
>>601544
Все, что измеряет.
Аноним 18/09/20 Птн 17:34:53 601547402
>>601544
Наблюдатель — это лично ты. Пока ты не спросил у мужика в белом халате, открывшим ящик с котом Шрёдингера, чё там случилось с котом, для тебя и кот находится в суперпозиции, и мужик в халате находится в суперпозиции (мужика, увидевшего мёртвого кота, с мужиком, увидевшим живого кота).
Аноним 18/09/20 Птн 17:42:13 601548403
>>601547
Сложна. Мужик ведь и спиздеть может.
Аноним 18/09/20 Птн 17:44:02 601549404
>>601543
Лифт на луне не построишь, а вот рельсотрон для вывода - самое то.
Аноним 18/09/20 Птн 17:44:55 601550405
>>601549
>Лифт на луне не построишь
Почему это?
Аноним 18/09/20 Птн 17:46:14 601551406
>>601550
Потому, что скорость, она приливно замкнута с землей, у нее нет ГСО.
Аноним 18/09/20 Птн 17:49:17 601552407
>>601551
Типа только точки L1/L2, нестабильные?
Аноним 18/09/20 Птн 17:50:05 601553408
>>601543
На безатмосферных планетах - да.
Аноним 18/09/20 Птн 17:52:51 601554409
image.png 518Кб, 640x442
640x442
image.png 881Кб, 942x608
942x608
>>601552
Да, нестабильные.
Отсутствие атмосферы на луне позволяет невозбранно разгоняться до второй космической с помощью рельсы или гаусса не боясь сгореть об атмосферу как на сраной земляшке, потому маняфантазии как раз о таком способе, а не лифте.
Космический фонтан и петля Лофстрома, кстати, тоже смогут работать, почему-то все про лифт знают, а про них нет.
Аноним 18/09/20 Птн 17:54:07 601555410
>>601544
Именно так. Ты можешь подставлять "взаимодействие" вместо "наблюдения" и смысл не изменится, больше того так оно и задумано. Наблюдатель - это издержки запутанной терминологии, наподобие "чёрной дыры" или "цвета кварков".

>>601547
В конечном счёте это сводится к взаимодействию с объектом эксперимента (и влиянием на него) при помощи какой-либо фундаментальной силы.
Аноним 18/09/20 Птн 17:57:20 601556411
>>601554
А что если с Земли откачать атмосферу, чтобы в космос было проще летать? Человеков переселим в купола и выдадим скафандры для прогулок.
Аноним 18/09/20 Птн 17:59:33 601557412
>>601554
Получается, для Венеры тоже нихуя не выйдет, пушо она очень медленно вращается? Чем быстрее вращается планета, тем ниже ГСО и короче лифт?
Аноним 18/09/20 Птн 17:59:38 601558413
>>601556
Не стоит лезть в эту тему. Просто не стоит.
Аноним 18/09/20 Птн 18:00:07 601560414
>>601556
Тащемта в космос проще летать, чем планетарную атмосферу откачивать.
Аноним 18/09/20 Птн 18:01:49 601561415
>>601555
В конечном счёте взаимодействие с объектом эксперимента и является ключевой вещью, а разумность/неразумность наблюдателя вообще ни при чем.
Аноним 18/09/20 Птн 18:03:25 601563416
>>601561
Ну да, про то и речь.
Хотя есть дяденька по фамилии Пенроуз, который много набрасывал говна на вентилятор в этом плане, но всё это без каких-либо оснований.
Аноним 18/09/20 Птн 18:05:22 601564417
>>601547
>>601555
То есть не "спросил", а провзаимодействовал? Типа, если кот Шрёдингера находится на орбите Плутона, для меня он будет в суперпозиции до тех пор, пока оттуда не прилетит какой-нибудь случайный фотон именно в меня (или в то, с чем я быстро провзаимодействую)?
Аноним 18/09/20 Птн 18:07:42 601565418
>>601564
Конечно. Инфа передаётся взаимодействием, а то не может передаваться быстрее с.
Аноним 18/09/20 Птн 18:08:47 601566419
image.png 4Кб, 158x113
158x113
image.png 74Кб, 2283x719
2283x719
>>601557
μ = GM
Радиус по формуле на пикриле считается. Он не должен выходить за сферу Хилла.

>Чем быстрее вращается планета, тем ниже ГСО и короче лифт?
А ни на какой другой планете невозможна геостационарная орбита. Только на Земле.
Аноним 18/09/20 Птн 18:11:56 601568420
>>601566
>ни на какой другой планете невозможна геостационарная орбита
Не придирайся, там вон выше было про ГСО вокруг Луны. Есть какой-то общий термин?
Аноним 18/09/20 Птн 18:13:31 601569421
>>601557
>>601566
Есть на Венере стационарная орбита. Но очень-очень далеко, неюзабельна для подобных целей.

>>601568
Стационарная орбита, без гео-.
Аноним 18/09/20 Птн 18:14:48 601571422
>>601569
>без гео-
Слава богу, хоть тут обошлись без апоариев и перийовиев.

А как быть с географией и геологией?
Аноним 18/09/20 Птн 18:16:24 601573423
Стикер 255Кб, 500x500
500x500
>>601568
Тащемта стационарная орбита.
И нет, >>601551
>у нее нет ГСО
Заметь, всё технически верно.

>>601569
Она почти выходит за сферу хилла, мы 40 тысяч километров-то не уверены что можем построить, а там почти миллион нужен.
Странно, мне казалось, что я упомянул сопромат после хилла.

>>601571
Не обошлось, у марса ареоцентрическая.
Джуно летает через перйобий.
Аноним 18/09/20 Птн 18:18:39 601574424
15570949619760.jpg 9Кб, 215x215
215x215
>>601571
На самом деле не обошлось, геспероцентрическая она.

Ареография и ареология - вполне себе термины, значит и гесперо-чё-то-там тоже.
Аноним 18/09/20 Птн 18:20:26 601575425
>>601574
А каковы последние новости гигеологии и талографии?
Аноним 18/09/20 Птн 18:24:04 601576426
>>601573
>>601574
АСТРОНАВТ
@
КОСМОНАВТ
@
ТАЙКУНАВТ


Тупые вопросы:
1) как оно будет звучать в Ботсване?
2) почему оно не "космочеловек" в каждом языке по-своему?

Ебучая лингвистика.
Аноним 18/09/20 Птн 18:25:01 601577427
Как относитесь к Юрию Кубареву? Годный дедок, или уже маразмеет на обидках? Думаю, не запилить ли отдельный тред по ЭРД.
Аноним 18/09/20 Птн 18:28:16 601578428
>>601576
ТАК ПОВЕЛОСЬ
К сожалению.
По-хорошему, все космонавты, т.к. к звездам не летаем.
Тайконавт это в англоязычной литературе для определения косманов из Поднебесной. Сами они называют своих огурцов трупами героями hángtiān yuán (航天员), а огурцов в целом Yǔ háng yuán (宇航员), что в целом по смыслу означает "персонал по работе в космическом пространстве".
Аноним 18/09/20 Птн 18:29:01 601579429
Аноним 18/09/20 Птн 18:30:01 601580430
>>601573
>мы 40 тысяч километров-то не уверены что можем построить
Ю вот? Акацуки на немногим меньше полуляма отходила, когда зацепилась за Венеру. Так-то да, стационарка там должна быть не особенно стабильной, и возмущения других тел там должны сильно сказываться. Но она есть, технически.
Аноним 18/09/20 Птн 18:31:08 601581431
>>601580
>Ю вот?
СОПРОМАТ
О
П
Р
О
М
А
Т
Аноним 18/09/20 Птн 18:31:44 601582432
>>601578
>Yǔ háng yuán (宇航员)
>персонал по работе в космическом пространстве
Космолюди. Нормально же, хоть кто-то додумался.
Аноним 18/09/20 Птн 18:32:26 601583433
1568339985072.jpg 25Кб, 542x542
542x542
>>601581
Каким боком тут сопротивление материалов?
Аноним 18/09/20 Птн 18:32:58 601584434
>>601583
Прямым. Ты не можешь построить лифт на миллион километров.
Аноним 18/09/20 Птн 18:33:25 601585435
>>601584
А, ты про лифт всё, тут конечно нет базара.
Аноним 18/09/20 Птн 18:46:00 601587436
>>601583
Это тот самый кот Шредингера?
Аноним 18/09/20 Птн 19:39:18 601596437
>>601587
Это та самая "собака сутулая".
Аноним 18/09/20 Птн 19:42:29 601598438
image.png 1260Кб, 750x920
750x920
>>601587
>>601596
Кот Шредингера
Собака Павлова
Лошадь Пржевальского
А огурец чей?
Аноним 18/09/20 Птн 19:49:31 601599439
>>601565
А если записку с помощью Альбукерке передать, будет считаться что инфа быстрее с передается?
Аноним 18/09/20 Птн 19:55:28 601601440
>>601599
По-моему да, если по диаграмме пенроуза посмотреть, то ты сжимая пространство искривляешь мировые линии которые в итоге могут превышать с.
Интересно, кто-нибудь рассматривал как альбукеркой внутрь черной дыры заглядывать?
Аноним 18/09/20 Птн 20:07:56 601602441
image.png 6105Кб, 2158x1256
2158x1256
>>601598
Доктора Попова же
Аноним 18/09/20 Птн 20:09:56 601603442
image.png 119Кб, 208x316
208x316
Аноним 18/09/20 Птн 20:43:37 601612443
>>601547
Да там сам кот сколапсирует/сдекогерирует (в зависимости от интерпретации) состояние того несчастного неустойчивого атома до всяких прочих наблюдателей. А скорее, до кота это сделает фотоумножитель. Так что кот мертвоживых котов не будет даже в теории.
Аноним 18/09/20 Птн 22:02:32 601620444
Чтобы определить примерный состав (верхнего слоя) какой-то металлической поверхности достаточно на нее посмотреть спектрографом? Нужно ли светить какими-то хитрыми спектрами, или обычный дневной свет сгодится?
Аноним 18/09/20 Птн 22:03:27 601621445
>>601612
>не знает про квантовую луну
Так ты Око Вселенной никогда не найдёшь.
Аноним 18/09/20 Птн 22:44:56 601625446
что такое энергетическая щель сверхпроводника?
Аноним 18/09/20 Птн 23:07:35 601628447
>>601621
Субъективному идеализму нет места в науке. Так Вы не Око Вселенной найдёте, а другое, хотя и созвучное её место. Даже математики, те, что склонны к идеализму, страдают его объективной, а не субъективной версией.
Аноним 19/09/20 Суб 02:37:09 601643448
i(70).jpg 46Кб, 720x696
720x696
>>601598
Огурец дальтоника, очевидно жи.
Аноним 19/09/20 Суб 02:49:15 601645449
>>601503
>А если сетка будет мелкая, как плетение на ткани, сможет ли она контрить УФ?
Да, если размер ячейки будет сравним с длиной волны.
Аноним 19/09/20 Суб 02:52:19 601646450
>>601545
Кроме электромагнитного подвеса ничего и не придумаешь, не колесами же 40 тыс км наматывать. Насчет утолщающейся толщины неизвестно, может там вообще ленты будут.
Аноним 19/09/20 Суб 02:54:59 601647451
>>601551
>она приливно замкнута с землей, у нее нет ГСО.
Вращение вокруг оси и соответственно центробежная сила есть же.
Аноним 19/09/20 Суб 04:10:56 601649452
>>601646
Там же вроде нагрузка растет что дурная. Вряд ли там линейным увеличением сечения можно отделаться, если конечно не делать лифт из анобтаниума.
Аноним 19/09/20 Суб 05:41:20 601650453
>>601647
Стационарная орбита у такого тела вдалеке от массивных тел была бы на высоте 90 000 км. Только сфера хилла Луны 60 000км. У нее нет орбиты на высоте 90 000км. Объект "выведенный" на эту "орбиту" ебнется оземь или будет вышвырнут землею.
Аноним 19/09/20 Суб 05:43:44 601651454
А че никто вообще не рассматривает пусковую петлю или космический фонтан? Они не требуют анобтаниумов, почему бы об этих мегаструктурах не говорить?
Аноним 19/09/20 Суб 08:32:03 601654455
>>601651
Может быть потому, что петля легковоспламеняема, а фонтан похож на пенис?
Аноним 19/09/20 Суб 08:33:14 601655456
image.png 95Кб, 300x300
300x300
Аноним 19/09/20 Суб 09:15:48 601656457
>>601650
>На Луне стационарной орбиты как таковой нет, однако для строительства лифта можно использовать точки Лагранжа L1 и L2 (которые в небе Луны почти неподвижны), при этом основание лифта должно находиться в центре видимой или обратной стороны Луны соответственно.
Если верить википедии.
Аноним 19/09/20 Суб 09:18:41 601657458
>>601656
И постоянные расходы на активную стабилизацию, чтоб эта ебала не намоталась на луну.
Не, серьезно, на ней можно рельсотрон строить и разгонять объекты до второй космической без проблем, атмосферы же нет, это простое и крайне энергоэффективное средство.
Аноним 19/09/20 Суб 12:31:05 601665459
image.png 907Кб, 641x684
641x684
Аноним 19/09/20 Суб 17:25:09 601688460
>>601643
Я гей, но тест говорит что я слабоумный шизофреник эпилептикч но не гей.
Аноним 19/09/20 Суб 20:19:03 601713461
>>601712
Не, для таких целей считай что равномерно. Гораздо сильнее мимопролеталы влияют.
Аноним 19/09/20 Суб 21:53:03 601730462
Аноним 19/09/20 Суб 21:54:40 601732463
image.png 65Кб, 225x225
225x225
Аноним 20/09/20 Вск 01:36:25 601757464
1600558362529.jpg 122Кб, 800x427
800x427
Допустим что кротовые норы возможны, как создать кротовую нору? Как задать координаты места в котором будем инициировать кротовую нору?
Аноним 20/09/20 Вск 02:32:17 601759465
>>601757
Проблема их в том, что они предполагают сверхсветовое перемещение и путешествие во времени. Как-то всерьёз заниматься тем, что так кардинально рушит причинность вряд ли кто-то занимается.
А сами червоточины - продукт математики, в теории их создание возможно с помощью экзотической материи, что означает в переводе на русский:
А ХУЙ ЕГО ЗНАЕТ
Аноним 20/09/20 Вск 03:16:11 601763466
>>601759
Хотя бы вики чекни, даже там пишут что червоточины не противоречат ото.
Аноним 20/09/20 Вск 03:20:14 601764467
>>601763
Где я писал, что они противоречат ОТО?
Я наоборот пишу, что они именно ей и выведены. Математически.
ОТО не говорит ничего о причинности.
Аноним 20/09/20 Вск 03:34:00 601765468
>>601764
А вот координаты другой стороны наверное бы зависели от координатов точки где бы зарождалась эта самая нора и от энергии переданной той самой "экзотической" материи с отрицательной плотностью энергии...
Аноним 20/09/20 Вск 03:40:28 601766469
>>601765
Математически... Наверное.
Проблема тут в том, что физически для этого нет идей.
Пузырь альбукерки тоже математически работает, и даже с современными выкладками требует "всего лишь" энергии всей массы Юпитера, а не Вселенной, но интерферометр Уайта у нас до сих пор его не надул даже на миллионную как там у нас в шапке.
Мы сейчас обсуждаем самый фронтир и даже за него заходим, куда прикладная физика ещё не ступала.
Если будешь в курсе и почитывать именно публикации и отзывы, то увидишь, что каких-то колоссальных чудес нет.
Что-то особо офигенное и удивительное, что только сейчас в прессе появляется, было подтверждено раньше, а было найдено экспериментально еще раньше, а предсказано вообще десятилетиями ранее.
Новые интересные вещи я ожидаю от ЛИГО, БАК и французов (забыл университет).
Новые данные по гравиволнам, темной материи, аксионам и металлическому водороду.

Кстати, где там ЛИЗА, такая-то йоба должна была бы быть, такой бонус к ЛИГО и ВИРГО.
И это, в СНГ или Китае не планируют гравитационные обсерватории?
Аноним 20/09/20 Вск 03:42:37 601767470
>>601766
Да у России и база на Луне то ли уже есть, то ли будет в ближайшие годы. Ну ты понел.
Аноним 20/09/20 Вск 03:46:08 601768471
>>601767
Нет, это не ко мне.
Не знаю, зачем ты это написал.
Аноним 20/09/20 Вск 03:47:32 601769472
>>601768
Хотел намекнуть, что планы Рашки на недешевые научные проекты можно смело игнорировать.
Аноним 20/09/20 Вск 03:52:39 601770473
>>601769
Я не одну "рашку" упомянул, почему ты именно на ней стал внимание акцентировать?
Китай может себе позволить крупную обсерваторию, и, собственно запилили четыре года назад вторую по апертуре в мире радиообсерваторию (и крупнейшую тарелку) https://en.wikipedia.org/wiki/Five-hundred-meter_Aperture_Spherical_Telescope. Первая по апертуре радиообсерватория, кстати, в этой самой "рашке", на 600 метров.
На западном полушарии всё хорошо с обсерваториями, мне интересно каково с ними, по крайней мере, в планах, в полушарии восточном. Это огромный буст к науке, и неважно кто, где и на чьи деньги это строит, если строит.
Аноним 20/09/20 Вск 04:02:09 601771474
>>601770
>>601766
Забейте, кстати, я сам все прочитал и нашёл.
В Индии ближайший хз когда.
ЛИЗА сильно в 30-х годах.
Китайцы планируют космический интерферометр в 30-х.

А ЛИГО и ВИРГО с нулевых работали в первом поколении, были улучшены после 2015 до второго поколения, и тогда и пошли подтвержденные наблюдения слияния массивных объектов.
В России не планируется.
Аноним 21/09/20 Пнд 00:07:17 601834475
>>601620
Бамп. Если на Красной площади приземлится корабль инопланетян, можно просто на него посмотреть, чтобы прикинуть состав обшивки?
Аноним 21/09/20 Пнд 00:38:40 601837476
1-alien-planets[...].jpg 156Кб, 900x598
900x598
Space6[1].jpg 214Кб, 1280x831
1280x831
157664375154-83[...].jpg 2950Кб, 2880x1920
2880x1920
scale1200[1] 101Кб, 1200x675
1200x675
Вопрос на стыке не только тредов, но даже разделов. Однако, слава богу это ТТВ, тут ответят нормально.
Существует ли в природе ФЕНТЕЗИ, действие которой происходит на другой планете в чужой солнечной системе, чтобы там была классическая фентезятина с магией и средними веками, но твердотой в плане описания этой планетарной системы. Тут есть несколько лун? Описано, по каким орбитам они ходят, как влияют и почему. На планете иные распределения сезонов? Почему? Как обосновано?
Я просто не знаю, как правильно искать, чтобы была такая, знаете, сказочная фентезя, но при этом каменная твердота в описании мироздания.
Может, вы читали фентези, в котором главгерой был вынужден изучать астрономию и географию?
Аноним 21/09/20 Пнд 01:55:54 601849477
Аноним 21/09/20 Пнд 03:06:22 601853478
>>601837
Не читал, но осуждаю.

>с магией и средними веками, но твердотой в плане описания этой планетарной системы
В Средние века на Земляшке нихуя про СС не знали. Коперник это уже в Новое время. Гелиоцентрика, в принципе, еще в античности возникла, но в суровые Средние века по ней не угорали (разве что на кострах). Да и минимум знаний раннего Нового времени на твердоту не тянет. О какой-то твердоте можно говорить разве что с XIX века, наверное, когда всякие Роши завелись. Да и то, что там за пиздец на Венере и про отсутствие каналов на Марсе только в XX узнали.

Короче, фентезятине это ни к чему, как по мне, особенно если там магия.
Аноним 21/09/20 Пнд 03:19:40 601855479
Аноним 21/09/20 Пнд 03:23:39 601856480
>>601855
Как там плоский мир затвердел?
Аноним 21/09/20 Пнд 03:43:38 601857481
>>601856
Читай, там чуть ли не в каждой книге как мир устроен.
Аноним 21/09/20 Пнд 03:45:56 601858482
>>601857
Нехуй делать, я комиксы читать устаю. Сомневаюсь, что там "каменная твердота". Предел прочности гранита учтен?
Аноним 21/09/20 Пнд 03:47:10 601859483
Аноним 21/09/20 Пнд 03:47:21 601860484
>>601858
А я наоборот, не могу в комиксы, а книги как нехуй.
Аноним 21/09/20 Пнд 03:47:48 601861485
>>601859
Я лучше киношку пасмарю.
Аноним 21/09/20 Пнд 11:39:48 601915486
>>601857
>>601858
"Ну это всё магия короче." Охуенная твердота.
Аноним 21/09/20 Пнд 13:28:19 601943487
>>601915
>"Ну это всё магия короче."
ТВЁРДО И ЧЁТКО?
Аноним 21/09/20 Пнд 14:34:46 601964488
image.png 3063Кб, 1600x1000
1600x1000
Аноним 21/09/20 Пнд 22:05:09 602056489
>>601770
>Первая по апертуре радиообсерватория, кстати, в этой самой "рашке", на 600 метров
Апертура-то там не цельная, собирающая поверхность у него эквивалентна 130 кажется метровой тарелке. Для апертуры есть РСДБ.
Аноним 21/09/20 Пнд 22:06:03 602057490
>>601943
МЯГКО И РАСПЛЫВЧАТО
Аноним 21/09/20 Пнд 22:11:04 602060491
>>601771
>А ЛИГО и ВИРГО с нулевых работали в первом поколении, были улучшены после 2015 до второго поколения, и тогда и пошли подтвержденные наблюдения слияния массивных объектов.
На самом деле LIGO делался 40 лет. Это гигантская работа Торна и причастных, вопреки снобам из MIT положившим жизни на создание этого инструмента и нового направления в астрономии, и добившихся успеха. Читни-ка книжца под названием Gravity's Kiss от Гарри Коллинза.
Аноним 21/09/20 Пнд 22:11:25 602061492
>>601620
Нужно испарить кусочек.
Аноним 21/09/20 Пнд 22:13:19 602062493
>>602061
А почему с металла в твердой фазе нельзя снять спектр? Или он недостаточно информативный?
Аноним 21/09/20 Пнд 22:16:24 602065494
>>602062
Потому что он не излучает свет своего спектра. А отражённый будет хер знает от чего зависеть - от спектра падающего (переотражённого с соседних объектов), от формы поверхности даже.
Аноним 21/09/20 Пнд 22:17:58 602066495
>>602062
Можно, если просветить очень тонкий слой его калиброванным источником света. Просто посмотрев на подсвеченный объект - не получится.
Аноним 21/09/20 Пнд 22:21:30 602070496
thinking emoji [...].gif 455Кб, 256x256
256x256
>>602065
Но как форма поверхности может влиять на спектр?
Аноним 21/09/20 Пнд 22:21:34 602071497
>>602065
То есть отраженный спектр состоит из малопонятной хуйни, а спектр света, прошедшего через облачко, - это спектр источника минус линии поглощения металла?
Аноним 21/09/20 Пнд 22:22:39 602073498
Аноним 21/09/20 Пнд 22:27:08 602077499
Аноним 21/09/20 Пнд 22:32:03 602080500
>>602073
Спасибо. Но так или иначе, ссылки на "неизвестный металл" во всяких нф-фильмах это лажа, ведь достаточно поскрести его алмазом и кинуть пыль в костер, так?
Аноним 21/09/20 Пнд 22:32:50 602082501
>>602071
Нет, спектр излучения раскалённой капли - это собственный спектр. Его и можно замерить. Нужно, чтобы сам материал излучал свет в нагретом состоянии.

Можно померить и спектр света, прошедшего через облачко, если:
- исходный луч обладает известными спектральными характеристиками
- он не монохромный (как из многих лазеров например), а имеет в себе все длины волн в какой-то степени
- ты уверен что поймал только твой луч через облачко, без дополнительной херни.
Химические спектрометры зачастую так и работают, пихаешь раствор в кювету, суешь внутрь, и он измеряется на просвет. Но для этого нужны очень контролируемые и предсказуемые условия.
Аноним 21/09/20 Пнд 22:37:06 602084502
>>602080
Тут всё тоже не так просто. Спектр - это именно что спектр, а не химический состав материала. Ты лишь можешь сравнить линии спектра с тем, что уже известно тебе. Кроме того, "неизвестный инопланетный металл" может оказаться сложным композитом, иметь какую-нибудь хитрую структуру и т.п., в общем химический состав, даже если его можно получить, может ни о чём не сказать. Тут надо снимать не только спектр, а полноценно реверс-инжинирить материал целиком - рентгеноскопия, электронная микроскопия и куча разных техник. И это может быть нетривиальной и небыстрой задачей.
Аноним 21/09/20 Пнд 22:41:49 602086503
>>602084
> Ты лишь можешь сравнить линии спектра с тем, что уже известно тебе.
А для молекул он как-то вычисляется по спектру атомов? Ну то есть, мы можем предсказать спектр химического соединения, которое мы пока не получали? Или еще лучше, по неизвестному спектру выяснить примерную химическую формулу, даже если мы с ней не сталкивались? Или так не получится?
Аноним 21/09/20 Пнд 22:59:34 602097504
>>602086
>А для молекул он как-то вычисляется по спектру атомов?
Зависит от условий. Если молекулы конкретного вещества не распадаются, будучи раскалёнными - можно получить спектр всей молекулы.
>мы можем предсказать спектр химического соединения, которое мы пока не получали
Вывести спектр по формуле - да, конечно, сравнительно без проблем.
Вывести неизвестную формулу по спектру - значительно сложнее, гораздо проще если всё промерено дополнительными техниками. Спектрометрия разных сортов и мастей - это только писечка на тортике, там очень дохуя всего.

В любом случае, чтобы анализировать любое неизвестное вещество, нужен его кусочек в строго контролируемых лабораторных условиях. А повторить его - уже совсем иная задача, может быть ещё менее тривиальной.
Аноним 21/09/20 Пнд 23:04:08 602100505
>>602097
А есть хоть какие-нибудь указания на существование стабильной экзотики типа мюония? Ну, типа, одиночный нейтрон же вон тоже нестабилен, а если его тщательно перемешать с протонами - то вполне себе. Существуют ли какие-нибудь гипотезы насчет получения стабильных атомных ядер с чем-то еще кроме протонов и нейтронов? Это к вопросу о получении загадочного спектра, который ни на что не похож.
Аноним 21/09/20 Пнд 23:05:10 602101506
>>602100
Охуенный вопрос, присоединяюсь к любопытствующему.
Аноним 22/09/20 Втр 00:17:04 602112507
Как вы не сретесь от космоса? Есть программы типо спейс энджина, или на телефон обзор неба, но все юзают их как ни в чем не бывало, для меня это тяжелейшая фобия, от размеров вселенной в дрожб бросает и я не понимаю почему это никого не пугает, даже картинки с планетами страшно открывать в 4к разрешении.
Аноним 22/09/20 Втр 00:19:26 602114508
>>602112
А подкроватных монстров не боишься?
Аноним 22/09/20 Втр 00:19:35 602115509
>>602112
Пугало когда было лет десять.
Чем больше узнаешь и больше понимаешь, тем меньше боишься, по крайней мере так у меня было.
Зайди в космических страхов тред, поделись своими ужасами.
Я хоть и не боюсь космоса, но пару раз были офигенные кошмары про падение земли и меня с нее на газовый гигант.

Любопытство и знание пересиливает страх.
Аноним 22/09/20 Втр 00:40:04 602119510
А фториды кислорода рассматривались для окислителя?
Они такие же пиздецовые как сам фтор или проще? А в энергетическом плане?
Аноним 22/09/20 Втр 01:04:24 602123511
Kuiperp.jpg 135Кб, 1318x1318
1318x1318
Аноним 10/11/20 Втр 19:55:45 611834512
>>602119
>рассматривались для окислителя?
если создашь тему про топлива, с кратким перечислением самых энергоёмких. То приму участие с числами в детальном обсуждении каждого из них.
Тут лишь уточню, что в лучших химических топливах водород не сгорает, а является инертным наполнителем, понижающим температуру пламени.
Настройки X
Ответить в тред X
15000
Макс объем: 40Mб, макс кол-во файлов: 4
Кликни/брось файл/ctrl-v
Стикеры X
Избранное / Топ тредов